Download as pdf or txt
Download as pdf or txt
You are on page 1of 121

Lecture Notes

BBA120 Business Mathematics

2018

BBA120 Business Mathematics:The course aims to provide an


introduction to mathematical concepts and lay down a foundation for
applications of basic tools and techniques for various areas of business
such as economics, accountancy and the life and social sciences. It
begins with non calculations topics as Basic mathematics, equations,
functions, matrix algebra, mathematics of finance etc. Then it progress
through both single-variable and multi-variable calculus. An abundance
and variety of applications appear throughout the course. Students
continually see how the mathematics they are learning can be applied
to practical business problems. These applications over such diverse
areas as business, economics, sociology, finance etc.

Bonaventure Chipeta.
School of Business and Economics
Department of Business Administration
Email:bonveee@gmail.com
Preface

These lecture notes are for the course BBA120 “Business Mathematics” for second semester
2018 intake at the University of Lusaka.

The author wishes to acknowledge that these lecture notes are collected from the references
listed in Bibliography, and from many other sources the author has forgotten. The author
claims no originality, and hopes not to be sued for plagiarizing or for violating the sacred
copyright laws.

6th August 2018


BBA 120 Business Mathematics

Contents
Unit 1: Mathematical Preliminaries 3
1.1 Set Theory ................................................................................................................... 3
1.1.1 Sets and elements ........................................................................................... 3
1.1.2. Specification of sets....................................................................................... 4
(a) List notation. ............................................................................................. 4
(b) Set builder notation. ................................................................................. 4
(c) Recursive rules. ........................................................................................ 4
1.1.3. Identity and cardinality .................................................................................. 5
1.1.4. Subsets ........................................................................................................... 5
1.1.5. Power sets ...................................................................................................... 5
1.1.6. Operations on sets: union, intersection.......................................................... 5
1.1.7 More operations on sets: difference, complement .......................................... 7
1.1.8 De Morgan's laws ........................................................................................... 8
1.1.9 Associative and Distributive laws of Set Operations ..................................... 9
Exercise 1.1 : Sets and Subsets ............................................................................. 10
Exercise 1.2 : Set Operations ................................................................................ 11
1.2.0 Common Number Sets ........................................................................................... 13
1.2.1 Intervals ........................................................................................................ 16
Inequalities ................................................................................................... 17
Interval Notation .......................................................................................... 17
Number Line ................................................................................................ 18
Open or Closed Intervals ............................................................................. 19
Intervals To Infinity (but not beyond!) ........................................................ 19
Exercises 2.1 .......................................................................................................... 20
BBA 120 Business Mathematics

Unit 1: Mathematical Preliminaries


Many models and problems in modern economics and finance can be
expressed using the language of mathematics and analyzed using
mathematical techniques.
The next few sections are a review of important material that you studied
in high school algebra, trigonometry and pre-calculus courses ant that is
essential prerequisite for studying calculus.
The aim throughout this Unit is to show how a range of important
mathematical techniques work and how they can be used to explore and
understand the structure of economic models.

1.1 Set Theory


Mathematics has its own terminologies and notation which is clear and
concise and enables us to carry out calculations. Set theory is the milieu
in which mathematics takes place today.

1.1.1 Sets and elements


Set theory is a basis of modern mathematics, and notions of set theory are
used in all formal descriptions.
Description: a set is a collection of objects which are called the members
Definition or elements of that set. If we have a set we say that some objects belong
sets consist of their elements.
A Set is a collection Examples: the set of students in this room; the English alphabet may be
of objects viewed as the set of letters of the English language; the set of natural
numbers1; etc.
So sets can consist of elements of various natures: people, physical
objects, numbers, signs, other sets, etc. (We will use the words object or
entity in a very broad way to include all these different kinds of things.)
The membership criteria for a set must in principle be well-defined, and
not vague.
Sets can be finite or infinite.
There is exactly one set, the empty set, or null set, which has no members
at all.
A set with only one member is called a singleton or a singleton set.
(“Singleton of a”)

3
4 Unit 1: Mathematical Preliminaries

Notation: A, B, C, … for sets; a, b, c, … or x, y, z, … for members. b ∈ A


if b belongs to A (B ∈ A if both A and B are sets and B is a member of A)
and c ∉ A, if c doesn’t belong to A.
∅ is used for the empty set.

1.1.2. Specification of sets


There are three main ways to specify a set:
a) by listing all its members (list notation);
b) by stating a property of its elements (Set builder notation);
c) by defining a set of rules which generates (defines) its
members (recursive rules).

(a) List notation.


The first way is suitable only for finite sets. In this case we list names of
elements of a set, separate them by commas and enclose them in braces:
Examples: {1, 12, 45}, {George Washington, Bill Clinton}, {a,b,d,m}.
Note that we do not care about the order of elements of the list, and
elements can be listed several times. {1, 12, 45}, {12, 1, 45,1} and
{45,12, 45,1} are different representations of the same set (see below the
notion of identity of sets).

(b) Set builder notation.


Example:
{x|x is a natural number and x < 8}
Reading: “the set of all x such that x is a natural number and is less than
8”
So the second part of this notation is a property the members of the set
share (a condition or a predicate which holds for members of this set).
Other examples:
{ x|x is a letter of Russian alphabet}
{y |y is a student of UMass and y is older than 25}
General form:
{ x|P(x)}, where P is some predicate (condition, property).

(c) Recursive rules.


Example – the set E of even numbers greater than 3:
a) 4 ∈ E
b) if x ∈ E, then x + 2 ∈ E
c) nothing else belongs to E.
The first rule is the basis of recursion, the second one generates new
elements from the elements defined before and the third rule restricts the
defined set to the elements generated by rules a and b. (The third rule
BBA 120 Business Mathematics

should always be there; sometimes in practice it is left implicit. It’s best


when you’re a beginner to make it explicit.)

1.1.3. Identity and cardinality


Two sets are identical if and only if 2 they have exactly the same
members. So A = B iff for every x, x ∈ A ⇔ x ∈ B.
For example, {0,2,4} = {x| x is an even natural number less than 5}
From the definition of identity follows that there exists only one empty
set; its identity is fully determined by its absence of members. Note that
empty list notation {} is not usually used for the empty set, we have a
special symbol ∅ for it.
The number of elements in a set A is called the cardinality of A, written
n(A) or |A|. The cardinality of a finite set is a natural number. Infinite sets
also have cardinalities but they are not natural numbers.

1.1.4. Subsets
A set A is a subset of a set B iff every element of A is also an element of
B. Such a relation between sets is denoted by A ⊆ B. If A ⊆ B and A ≠ B
we call A a proper subset of B and write A ⊂ B. (Caution: sometimes ⊂ is
used the way we are using ⊆.)
Both signs can be negated using the slash / through the sign.
Examples:
{a,b} ⊆ {d,a,b,e} and {a,b} ⊂ {d,a,b,e}, {a,b} ⊆ {a,b}, but {a,b} ⊄
{a,b}.
Note that the empty set is a subset of every set. ∅ ⊆ A for every set A.
Why?
Be careful about the difference between “member of” and “subset
of”;

1.1.5. Power sets


The set of all subsets of a set A is called the power set of A and denoted
as ℘(A). For example, if A = {a,b}, ℘(A) = {∅, {a}, {b}, {a,b}}.
From the example above: a ∈ A; {a} ⊆ A; {a} ∈ ℘(A)
∅ ⊆ A; ∅ ∉ A; ∅ ∈ ℘(A); ∅ ⊆ ℘(A)

1.1.6. Operations on sets: union, intersection.


We define several operations on sets. Let A and B be arbitrary sets.
The union of A and B, written A ∪ B, is the set whose elements are just
the elements of A or B or of both. In the predicate notation the definition
is
A ∪ B ={ x| x ∈ A or x ∈ B}

5
6 Unit 1: Mathematical Preliminaries

Examples. Let K = {a,b}, L = {c,d} and M = {b,d}, then


K ∪ L = {a,b,c,d} (K ∪ L) ∪ M = K ∪ (L ∪ M) = {a,b,c,d}
K ∪ M = {a,b,d} K∪K=K
L ∪ M = {b,c,d} K ∪ ∅ = ∅ ∪ K = K = {a,b}.
There is a nice method for visually representing sets and set-theoretic
operations, called
Venn diagrams. Each set is drawn as a circle and its members represented
by points within it. The diagrams for two arbitrarily chosen sets are
represented as partially intersecting – the most general case – as in Figure
1.1 below. The region designated ‘1’ contains elements which are
members of A but not of B; region 2, those members in B but not in A;
and region 3, members of both B and A. Points in region 4 outside the
diagram represent elements in neither set.

1 2
3

4
B
Figure 1.1

The Venn diagram for the union of A and B is shown in Figure 1.2. The
results of operations in this and other diagrams are shown by shading
areas.

B
Figure 1.2
The intersection of A and B, written A ∩ B, is the set whose elements are
just the elements of both A and B. In the predicate notation the definition
is
A ∩ B ={ x| x ∈ A and x ∈ B}
Examples:
K∩L=∅ (K ∩ L) ∩ M = K ∩ (L ∩ M) = ∅
K ∩ M = {b} K∩K=K
L ∩ M = {d} K ∩ ∅ = ∅ ∩ K = ∅.
BBA 120 Business Mathematics

The general case of intersection of arbitrary sets A and B is represented


by the Venn diagram of Figure 1.3. The intersection of three arbitrary sets
A,B and C is shown in the Venn diagram of Figure 1.4.

A
A

B C

Figure 1.3 Figure 1.4

1.1.7 More operations on sets: difference, complement


Another binary operation on arbitrary sets is the difference “A minus B”,
written A – B, which ‘subtracts’ from A all elements which are in B.
[Also called relative complement: the complement of B relative to A.] The
set builder’s notation defines this operation as follows:
A – B ={ x| x ∈ A and x ∉ B}
Examples: (using the previous K, L, M)
K – L = {a,b} K–K=∅
K – M = {a} K–∅=K
L – M = {c} ∅ – K = ∅.
The Venn diagram for the set-theoretic difference is shown in Figure 1–5.
Figure1.5:

A – B is also called the relative complement of B relative to A. This


operation is to be distinguished from the complement of a set A, written
A’, which is the set consisting of everything not in A. In predicate
notation
A’ = { x| x ∉ A}

7
8 Unit 1: Mathematical Preliminaries

It is natural to ask, where do these objects come from which do not


belong to A? In this case it is presupposed that there exists a universe of
discourse and all other sets are subsets of this set. The universe of
discourse is conventionally denoted by the symbol U.
Then we have
A’ =U – A
The Venn diagram with a shaded section for the complement of A is
shown in
Figure1.6

Figure 1.6

1.1.8 De Morgan's laws


In set theory, de Morgan's laws relate the three basic set operations
to each other; the union, the intersection, and the complement. De
Morgan's laws are named after the Indian-born British
mathematician and logician Augustus De Morgan (1806-1871).
If A and B are subsets of a set X , de Morgan's laws state that

(𝐴 ∪ 𝐵)′ = 𝐴′ ∩ 𝐵′

(𝐴 ∩ 𝐵)′ = 𝐴′ ∪ 𝐵′

Here, ∪ denotes the union, ∩ denotes the intersection, and A’ denotes


the set complement of A in X , i.e., A’=X-A .
Above, de Morgan's laws are written for two sets. In this form, they
are intuitively quite clear. For instance, the first claim states that an
element that is not in A∪B is not in A and not in B . It also states
that an elements not in A and not in B is not in A∪ B .
BBA 120 Business Mathematics

1.1.9 Associative and Distributive laws of Set Operations


Associative Law
The associative laws establish the rules of taking unions and
intersections of sets. They apply to all sets including the set
of real numbers.

𝐴 𝑈 (𝐵 𝑈 𝐶) = (𝐴 𝑈 𝐵) 𝑈 𝐶

This law states that taking the union of a set to the union of
two other sets is the same as taking the union of the original
set and one of the other two sets, and then taking the union
of the results with the last set.

𝐴 ∩ (𝐵 ∩ 𝐶) = (𝐴 ∩ 𝐵) ∩ 𝐶

This law states that taking the intersection of a set to the


intersection of two other sets is the same as taking the
intersection of the original set and one of the other two sets,
and then taking the intersection of the results with the last
set.

Distributive Law

The distributive laws also establish the rules of taking


unions and intersections of sets.
𝐴 𝑈 (𝐵 ∩ 𝐶) = (𝐴 𝑈 𝐵) ∩ (𝐴 𝑈 𝐶)

This law states that taking the union of a set to the


intersection of two other sets is the same as taking the union
of the original set and both the other two sets separately,
and then taking the intersection of the results.
𝐴 ∩ (𝐵 𝑈 𝐶) = (𝐴 ∩ 𝐵) 𝑈 (𝐴 ∩ 𝐶)

This law states that taking the intersection of a set to the


union of two other sets is the same as taking the intersection
of the original set and both the other two sets separately,
and then taking the union of the results.

9
10 Unit 1: Mathematical Preliminaries

Exercise 1.1 : Sets and Subsets


1. Let A={a, b, {c, d}, e}. How many elements does A contain?

2. Let A = {2, {4, 5}, 4}. Which statement is correct?


a) 5 is an element of A.
b) {5} is an element of A.
c) {4, 5} is an element of A.
d) {5} is a subset of A.
3. Which of these sets is finite?

a) {x | x is even}
b) {x | x < 5}
c) {1, 2, 3,...}
d) {1, 2, 3,...,999,1000}

4. Which of these sets is not a null set?


a) A = {x | 6x = 24 and 3x = 1}
b) B = {x | x + 10 = 10}
c) C = {x | x is a man older than 200 years}
d) D = {x | x < x}

5. Let S={1, 2, 3}. How many subsets does S contain?

6. Let D E. Suppose a D and b E. Which of the following


statements must be true?
a) c D
b) b D
c) a E
d) a D

7. Let A = {x | x is even}, B = {1, 2, 3,..., 99, 100}, C = {3, 5, 7, 9},


D = {101, 102} and E = {101, 103, 105}. Which of these sets
can equal S if S A and S and B are disjoint?

a) A b) B c) C d) D e) E

8. Let S = {a, b}. How many elements does the power set 2S
contain?

9. Which set S does the power set 2S = { , {1}, {2}, {3}, {1, 2},
{1, 3}, {2, 3}, {1, 2, 3}} come from?
a) {{1},{2},{3}}
b) {1, 2, 3}
BBA 120 Business Mathematics

c) {{1, 2}, {2, 3}, {1, 3}}


d) {{1, 2, 3}}

Exercise 1.2 : Set Operations


1. Let A = {x, y, z}, B = {v, w, x}. Which of the following statements
is correct?
a) A B = {v, w, x, y, z}
b) A B = {v, w, y, z}
c) A B = {v, w, x, y}
d) A B = {x, w, x, y, z}

2. Let A = {1, 2, 3, ..., 8, 9} and B = {3, 5, 7, 9}. Which of the


following statements is correct?.
a) A B = {2, 4, 6}
b) A B = {1, 2, 3, 4, 5, 6, 7, 8, 9}
c) A B = {1, 2, 4, 6, 8}
d) A B = {2, 4, 6, 8}

3. Let C = {1, 2, 3, 4} and D = {1, 3, 5, 7, 9}. How many elements


does the set C D contain?

How many elements does the set C D contain?

4. Let A = {2, 3, 4}, B = {3} and C = {x | x is even}. Which


statement is correct?
a) C A = B
b) C B = A
c) A C
d) C / A = B (Alternate notation for C-A)

5. Let A B, B C and D A = C. Which statement is always


false?
a) B D
b) A C
c) A=B
d) B D= and B A

11
12 Unit 1: Mathematical Preliminaries

6. What is shaded in the Venn diagram below?.


Set Theory
Applied to business
operations, set theory
can assist in planning
and operations.

Every element of
business can be
grouped into at least
one set such as
accounting, a) A B
management,
operations, production b) A B
and sales. c) A
d) B

Within those sets are


other sets. In
operations, for 7. What is shaded in the Venn diagram below?.
example, there are sets
of warehouse
operations, sales
operations and
administrative
operations.

In some cases, sets


intersect -- as sales
operations can
intersect the a) A B
operations set and the b) A'
sales set c) A-B
d) B-A

8. Let U = {1, 2, 3, ..., 8, 9} and A = {1, 3, 5, 7}. Find A'.


a) A' = {2, 4, 6, 8}
b) A' = {2, 4, 6, 8, 9}
c) A' = {2, 4, 6}
d) A' = {9}

9. Let U = {1, 2, 3,..., 8, 9}, B = {1, 3, 5, 7} and C = {2, 3, 4, 5, 6}.


How many elements does the set (B C)' contain?
BBA 120 Business Mathematics

How many elements does the set (C - B)' contain?

1.2.0 Common Number Sets

There are sets of numbers that are used so often that they have
special names and symbols:

13
14 Unit 1: Mathematical Preliminaries

Symbol
Description

Natural Numbers
The whole numbers from 1 upwards. (Or from 0 upwards in some fields of
mathematics).
The set is {1,2,3,...} or {0,1,2,3,...}

Integers
The whole numbers, {1,2,3,...} negative whole numbers {..., -3,-2,-1} and
zero {0}. So the set is {..., -3, -2, -1, 0, 1, 2, 3, ...}

(Z is for the German "Zahlen", meaning numbers, because I is used for the
set of imaginary numbers).

Rational Numbers

A rational number is any number that can be written as a ratio of two


integers. The set of rational numbers contains the set of integers since any
integer can be written as a fraction with a denominator of 1. A rational
number can have several different fractional representations. For example,
1/2 is equivalent to 2/4 or 132/264. In decimal representation, rational
numbers take the form of repeating decimals. Some examples of rational
numbers are:

Irrational Numbers
Any number that is not a Rational Number. These are numbers that can be
written as decimals, but not as fractions. They are non-repeating, non-
terminating decimals. Some examples of irrational numbers are:

Algebraic Numbers
BBA 120 Business Mathematics

Any number that is a solution to a polynomial equation with rational


coefficients.
Includes all Rational Numbers, and some Irrational Numbers.

Transcendental Numbers
Any number that is not an Algebraic Number
Examples of transcendental numbers include π and e.

Real Numbers
All Rational and Irrational numbers. They can also be positive, negative or
zero. They include the Algebraic Numbers and Transcendental Numbers.
A simple way to think about the Real Numbers is: any point anywhere on
the number line (not just the whole numbers).
Examples: 1.5, -12.3, 99, √2, π
They are called "Real" numbers because they are not Imaginary Numbers.

Imaginary Numbers
Numbers that when squared give a negative result.
If you square a real number you always get a positive, or zero, result. For
example 2×2=4, and (-2)×(-2)=4 also, so "imaginary" numbers can seem
impossible, but they are still useful!
Examples: √(-9) (=3i), 6i, -5.2i
The "unit" imaginary numbers is √(-1) (the square root of minus one), and
its symbol is i, or sometimes j.
i2 = -1

Complex Numbers
A combination of a real and an imaginary number in the form a + bi, where
a and b are real, and i is imaginary.
The values a and b can be zero, so the set of real numbers and the set of
imaginary numbers are subsets of the set of complex numbers.
Examples: 1 + i, 2 - 6i, -5.2i, 4

15
16 Unit 1: Mathematical Preliminaries

Together, the rational numbers and the irrational numbers form the set of Real Numbers.

Figure 2.1 The real number line

Intervals

1.2.1 Intervals
An Interval is all the real numbers between two given numbers. For
example, all the numbers between 1 and 6 is an interval, i.e. the set of all
numbers x satisfying 1 ≤ x ≤ 6 is an interval which contains 1 and 6, as
well as all numbers between them.
The interval 2 to 4 includes numbers such as:
BBA 120 Business Mathematics

2.1 2.1111 2.5 2.75 2.80001 π 7


/2 3.7937

And lots more!


Note that the boundary numbers may or may not be included in the
interval.
It isn't really clear.

Example: "An economy class ticket allows baggage of up to 20 kg in mass" If your bag is
exactly 20 kg ... will that be allowed or not?

I will show you how to be precise about this in each of three


popular methods:

 Inequalities
 The Number Line
 Interval Notation

Inequalities
In mathematics, an inequality is a relation that holds between two
values when they are different
Real numbers can be compared in size.

 The notation a < b means that a is less than b.


 The notation a > b means that a is greater than b.

In either case, a is not equal to b. These relations are known as


strict inequalities. The notation a < b may also be read as "a is
strictly less than b".
In contrast to strict inequalities, there are two types of inequality
relations that are not strict:

 The notation a ≤ b means that a is less than or equal to b (or,


equivalently, not greater than b, or at most b).
 The notation a ≥ b means that a is greater than or equal to b
(or, equivalently, not less than b, or at least b)

Example: x≤ 20 means ‘up to and including 20’

Interval Notation
In "Interval Notation" you just write the beginning and ending
numbers of the interval, and use:

17
18 Unit 1: Mathematical Preliminaries

 [ ] a square bracket if you want to include the end value, or


 ( ) a round bracket if you don't

Like this:

For example: (5, 12] Means from 5 to 12, do not include 5, but do
include 12

Number Line
With the Number Line you draw a thick line to show the values
you are including, and:

 a filled-in circle if you want to include the end value, or


 an open circle if you don't

Like this:
Example:

means all the numbers between 0 and 20, do not include 0, but do
include 20

Here is a handy table showing you all 3 methods (the interval is 1


to 2):

From 1 To 2

Not Including
Including 1 Not Including 1 Including 2
2

x≥1 x>1 x<2 x≤2


Inequality: "greater than "greater than" "less than" "less than
or equal to" or equal to"
BBA 120 Business Mathematics

Number line:

Interval notation: [1 (1 2) 2]

Example 2: "Competitors must be between 14 and 18"


So 14 is included, and "being 18" goes all the way up to (but not
including) 19.
As an inequality it looks like this:
14 ≤ Age < 19
On the number line it looks like this:

And using interval notation it is simply:


[14, 19)

Open or Closed Intervals


The terms "Open" and "Closed" are sometimes used when the end
value is included or not:

(a, b) a < x < b an open interval

[a, b) a ≤ x < b closed on left, open on right

(a, b] a < x ≤ b open on left, closed on right

[a, b] a ≤ x ≤ b a closed interval

These are intervals of finite length. We also have intervals of


infinite length.
Intervals To Infinity (but not beyond!)
We often use Infinity in interval notation.
Infinity is not a real number, in this case it just means "continuing
on ..."
Example: x greater than, or equal to, 3:
[3, +∞)

19
20 Unit 1: Mathematical Preliminaries

Note that we use the round bracket with infinity, because we don't
reach it!
There are 4 possible "infinite ends":

Interval Inequality

(a, +∞) x>a "greater than a"

[a, +∞) x≥a "greater than or equal to a"

(-∞, a) x<a "less than a"

(-∞, a] x≤a "less than or equal to a"

We could even show no limits by using this notation: (-∞, +∞)

Exercises 2.1
I. Write the inequality 4 ≤ x < 9 in interval notation
II. Write the inequality 4 ≥ x > -3 in interval notation
III. What inequality is defined in interval notation by (-∞, -2) ∪ [3, +∞)?
IV. What inequality is defined in interval notation by (-∞, 3] ∩ [1, +∞)?
V. The interval shown on the number line can be expressed as which
inequality:

a. x ≤ -5 or x > 4 b. x ≤ -5 and x > 4


c. -5 ≤ x < 4 d. -5 < x ≤ 4

VI.

Which of the following describes in interval notation the inequality

shown in the diagram?


BBA 120 Business Mathematics

a. (-∞, -2) ∪ [6, ∞) b. (-∞, -2] ∪ (6, ∞)


c. (-∞, -2) ∩ [6, ∞) d. (-∞, -2] ∩ (6, ∞)

VII. Determine the Truth of each statement. If the statement is false,


give a reason why that is so
1. -13 is an integer

−2
2. is rational
7

3. −3 is a positive integer

4. 0 is not rational

5. √3 is rational

6. √25 is not a positive integer


7
7. is a rational number
0

8. √2 is a real number

0
9. is rational
0

10. 𝜋 is a positive integer

11. -3 is to the right of -4 on the real- number line.

12. Every integer is positive or negative

13. 0 is a natural number

21
22 Unit 2: Equations and Inequalities

Unit 2: Equations and Inequalities

2.0 Arithmetic Operations


Arithmetic or arithmetic’s is
the oldest and most elementary
branch of mathematics, used by
almost everyone, for tasks
ranging from simple day-to-day
counting to advanced science
and business calculations. It
involves the study of quantity,
especially as the result of
operations that combine
numbers. In common usage, it refers to the simpler properties when using
the traditional operations of addition, subtraction, multiplication and
division with smaller values of numbers.

2.1. Addition and subtraction


Adding: If all the signs are the same, simply add all the terms and give
the answer with the common overall sign.

Subtracting: When subtracting any two numbers or two similar terms,


give the answer with the sign of the largest number or term.
If a and b are any two numbers, then we have the following rules
𝑎 + (−𝑏) = 𝑎 − 𝑏,
𝑎 − (+𝑏) = 𝑎 − 𝑏,
𝑎 − (−𝑏) = 𝑎 + 𝑏.
Thus we can regard −(−𝑏) as equal to +𝑏.
We consider a few examples:
4 + (−1) = 4 − 1 = 3,
and
3 − (−2) = 3 + 2 = 5.
The last example makes sense if we regard 3 − (−2) as the difference
between 3 and −2 on the number line.
Note that a − b will be negative if and only if a < b. For example,
−2 − (−1) = −2 + 1 = −1 < 0.
BBA 120 Business Mathematics

Examples

Add/subtract with numbers, mostly Add/subtract with variable terms


5 + 8 + 3 = 16 5𝑥 + 8𝑥 + 3𝑥 = 16𝑥
5 + 8 + 3 + 𝑣 = 16 + 𝑦 5𝑥 + 8𝑥 + 3𝑥 + 𝑣 = 16𝑥 + 𝑦
(The y-term is different, so it cannot be
5𝑥𝑣 + 8𝑥𝑣 + 3𝑥𝑣 + 𝑣 = 16𝑥𝑣 + 𝑦
added to the others)

2.2 Multiplication and Division

Multiplication
If a and b are any two positive numbers, then we have the following rules
for multiplying positive and negative numbers:
𝑎 × (−𝑏) = −(𝑎 × 𝑏),
(−𝑎) × 𝑏 = −(𝑎 × 𝑏),
(−𝑎) × (−𝑏) = 𝑎 × 𝑏.
Remember
So multiplication of two numbers of the same sign gives a positive
It is useful to remember number, while multiplication of two numbers of different signs gives a
that a minus sign is a -1, negative number.
so -5 is read as −1 × 5
For example, to calculate 2 × (−5), we multiply 2 by 5 and then place a
Also
minus sign before the answer. Thus, 2 × (−5) = −10.

0 × (any real number) = 0 These multiplication rules give, for example,


0 ÷ (any real number) = 0 (−2) × (−3) = 6, (−4) × 5 = −20, 7 × (−5) = −35.
But you cannot divide by
0 Division
The same rules hold for division because it is the same sort of operation
Brackets are used for as multiplication, since
grouping terms together 𝑎 1
in maths for: =𝑎×
𝑏 𝑏
(i) Clarity So the division of a number by another of the same sign gives a positive
(ii) Indicating the order in number, while division of a number by another of the opposite sign gives
which a series of a negative number.
operations should be
carried out
Examples
a) 5 × 7 = 35

23
24 Unit 2: Equations and Inequalities

b) −5 × −7 = 35
c) −5 × 7 = −35
d) (−15) ÷ (−3) = 5
e) (−16) ÷ 2 = −8
1
f) 2 ÷ (−4) = − 2
g) (−𝑥)(−𝑦) = 𝑥𝑦
h) 2(𝑥 + 4) = 2𝑥 + 8
i) (𝑥 − 3)(𝑥 + 4) = 𝑥(𝑥 + 4) − 3(𝑥 + 4)
= 𝑥 2 + 4𝑥 − 3𝑥 − 12
= 𝑥 2 + 𝑥 − 12

2.3 Order of Operations

The order in which operations in an arithmetical expression are


performed is important. Consider the calculation
12 + 8 ÷ 4.
Different answers are obtained depending on the order in which the
operations are executed.
If we first add together 12 and 8 and then divide by 4, the result is 5.
However, if we first divide 8 by 4 to give 2 and then add this to 12, the
result is 14.
Therefore, the order in which the mathematical operations are performed
Note: Other texts is important and the convention is as follows:
use the Acronyms Brackets, exponents, division, multiplication, addition, and subtraction.
BODMAS or
PEMDAS. This convention has the acronym BEDMAS.
However, the main point to remember is that if you want a calculation to
be done in a particular order, you should use brackets to avoid any
ambiguity.

Examples
Evaluate the expressions
a) 3 ∗ ( 5 + 8 ) − 22 ÷ 4 + 3
Brackets or Parenthesis first: 5 + 8 = 13
3 ∗ 13 − 22 ÷ 4 + 3
Exponent next: square the 2 or 22 = 4
3 ∗ 13 − 4 ÷ 4 + 3
BBA 120 Business Mathematics

Multiplication and Division next (3 ∗ 13) (4 ÷ 4)


left to right:
39 − 1 + 3
Addition and Subtraction next
left to right:
39 − 1 + 3 = 41

b) 23 × 3 + (5 − 1).

c) 4 – 3[4 – 2(6 – 3)] ÷ 2.

2.4 Fractions
𝑎
A fraction is a number that expresses part of a whole. It takes the form 𝑏
where a and b are any integers except that b ≠ 0.
The integers a and b are known as the numerator and denominator of
Note that in the the fraction, respectively.
𝑎
fraction 𝑏 Examples of statements that use fractions are,
a can be greater 3
 of students in a lecture may be female or
than b 5
1
 of a person’s income may be taxed by the government.
3

Fractions may be simplified to obtain what is known as a reduced


fraction or a fraction in its lowest terms. This is achieved by identifying
any common factors in the numerator and denominator and then
cancelling those factors by dividing both numerator and denominator by
them.
27
For example, consider the simplification of the fraction 45. Both the
numerator and denominator have 9 as a common factor since 27 = 9 × 3
and 45 = 9 × 5 and therefore it can be cancelled:
27 3 × 9 3
= =
45 9 × 5 5
27 3 3
We say that 45
and 5 are equivalent fractions and that 5
is a reduced
fraction.
Comparing two fractions
To compare the relative sizes of two fractions and also to add or subtract
two fractions, we express them in terms of a common denominator.
The common denominator is a number that each of the denominators of
the respective fractions divides, i.e., each is a factor of the common
denominator.

25
26 Unit 2: Equations and Inequalities

Suppose we wish to determine which is the greater of the two fractions


4 5
and .
9 11
Least Common
Multiple (LCM) The common denominator is 9 × 11 = 99. Each of the denominators (9
and 11) of the two fractions divides 99.
The least common
multiple (also called We follow a similar procedure when we want to add or subtract two
the lowest common fractions.
multiple or smallest
common multiple) of 2.4.1 Addition and Subtraction of Fractions
two integers a and b, To add or subtract fractions we use the following method;
usually denoted by
LCM(a, b), is the Step 1: Take a common denominator, that is, a number or term which is
smallest positive divisible by the denominator of each fraction to be added or subtracted. A
integer that is divisible safe bet is to use the product of all the individual denominators as the
by both a and b. common denominator.
If either a or b is 0, Step 2: For each fraction, divide each denominator into the common
LCM(a, b) is defined denominator, then multiply the answer by the numerator.
to be zero. Step 3: Simplify your answer if possible.
For example the LCM
of 4 and 6 is 12
Example
Simplify
13 5
a) − 16
Greatest Common 24
Divisor (gcd)
1 2 4
The greatest common b) +3−5
7
divisor (gcd), also
known as the greatest 𝑥 2𝑥 4𝑥
c) + −
common factor (gcf), 7 3 5
or highest common
factor (hcf), of two or 2.4.2 Multiplication and Division of fractions
more non-zero
To multiply together two fractions, we simply multiply the numerators
integers, is the largest
together and multiply the denominators together:
positive integer that
divides the numbers
without a remainder. 𝑎 c a × b ac
For example, the GCD × = =
of 8 and 12 is 4. 𝑏 d c × b bd

To divide one fraction by another, we multiply by the reciprocal of the


𝑏
divisor where the reciprocal of the fraction a/b is defined to be 𝑎 provided
a, b ≠ 0.

That is
𝑎 𝑐 a × d 𝑎𝑑
÷ = =
𝑏 𝑑 c × b 𝑐𝑏
BBA 120 Business Mathematics

Examples
2 5 (2)(5) 10
a) ( ) ( ) = =
3 7 (3)(7) 21

2 7 (−2)(7) 14
b) (− ) ( ) = =−
3 5 (3)(5) 15

2 3 2 (3)(2) 6 1
c) 3 × = ( ) ( ) = = =1
5 1 5 (1)(5) 5 5

3 (𝑥+3) 3(𝑥+3) 3𝑥+9


d) ( ) = = 2
𝑥 (𝑥−5) 𝑥(𝑥−5) 𝑥 −5𝑥

2
( ) 2 11 22
3
e) 5 = (3) ( 5 ) = 15
( )
11

2𝑥
𝑥+𝑦 2𝑥 2(𝑥−𝑦)
f) 3𝑥 = 𝑥+𝑦 3𝑥
2(𝑥−𝑦)

4𝑥(𝑥−𝑦) 4(𝑥−𝑦)
= = 3(𝑥+𝑦)
3𝑥(𝑥+𝑦)

2.5 Decimal Representation of Numbers

A fraction or rational number may be converted to its equivalent decimal


representation by dividing the numerator by the denominator.
3
For example, the decimal representation of 4 is found by dividing 3 by 4
to give 0.75. This is an example of a terminating decimal since it ends
after a finite number of digits.
The following are examples of rational numbers that have a terminating
decimal representation:

27
28 Unit 2: Equations and Inequalities

1 3
8
= 0.125, and 25
= 0.12.

Some fractions do not possess a finite decimal representation – they go


1
on forever. The fraction is one such example. Its decimal representation
In some text 3
books, a bar is 0.3333... where the dots denote that the 3’s are repeated and we write
notation is used 1
= 0. 3̇
instead of a dot 3
e.g.
52.71656565… where the dot over the number indicates that it is repeated indefinitely.
(65 repeating This is an example of a recurring decimal.
infinitely often) All rational numbers have a decimal representation that either terminates
may be written or contains an infinitely repeated finite sequence of numbers. Another
̅̅̅̅
52.7165 example of a recurring decimal is the decimal representation of 1/13:
1
= 0.0769230769230 … = 0.07̇69230̇
13
where the dots indicate the first and last digits in the repeated sequence.

2.5.1 Converting Decimals to Fractions


To convert a decimal to a fraction, you simply have to remember that the
first digit after the decimal point is a tenth, the second a hundredth, and
so on.
For example,
2 1
0.2 = =
10 5
and
375 3
0.375 = =
1000 8

2.5.2 Converting repeating decimals to fractions


When converting repeating decimals to fractions, just follow the two
steps below carefully.

Step 1: Let x equal the repeating decimal you are trying to convert to a
fraction

Step 2: Examine the repeating decimal to find the repeating digit(s)

Step 3: Place the repeating digit(s) to the left of the decimal point

Step 4: Place the repeating digit(s) to the right of the decimal point
BBA 120 Business Mathematics

Step 5: Subtract the left sides of the two equations. Then, subtract the
right sides of the two equations

As you subtract, just make sure that the difference is positive for both
sides

Now let's practice converting repeating decimals to fractions with two


good examples

Example #1:
What rational number or fraction is equal to 0.55555555555

Step 1: x = 0.5555555555

Step 2: After examination, the repeating digit is 5

Step 3: To place the repeating digit ( 5 ) to the left of the decimal point,
you need to move the decimal point 1 place to the right

Technically, moving a decimal point one place to the right is done by


multiplying the decimal number by 10.

When you multiply one side by a number, you have to multiply the other
side by the same number to keep the equation balanced

Thus, 10x = 5.555555555

Step 4: Place the repeating digit(s) to the right of the decimal point

Look at the equation in step 1 again. In this example, the repeating digit
is already to the right, so there is nothing else to do.

x = 0.5555555555

Step 5: Your two equations are:

10x = 5.555555555

x = 0.5555555555

10x - x = 5.555555555 − 0.555555555555

9x = 5

Divide both sides by 9

x = 5/9

29
30 Unit 2: Equations and Inequalities

Example #2:
What rational number or fraction is equal to 1.04242424242

Step 1: x = 1.04242424242

Step 2:After examination, the repeating digit is 42

Step 3:To place the repeating digit ( 42 ) to the left of the decimal point,
you need to move the decimal point 3 place to the right

Again, moving a decimal point three place to the right is done by


multiplying the decimal number by 1000.

When you multiply one side by a number, you have to multiply the other
side by the same number to keep the equation balanced

Thus, 1000x = 1042.42424242

Step 4:Place the repeating digit(s) to the right of the decimal point

In this example, the repeating digit is not immediately to the right of the
decimal point.

Look at the equation in step 1 one more time and you will see that there is
a zero between the repeating digit and the decimal point

To accomplish this, you have to move the decimal point 1 place to the
right

This is done by multiplying both sides by 10

10x = 10.4242424242

Step 5:Your two equations are:

1000x = 1042.42424242
BBA 120 Business Mathematics

10x = 10.42424242

1000x - 10x = 1042.42424242 − 10.42424242

990x = 1032

Divide both sides by 990:


x = 1032/990

To master this lesson about converting repeating decimals to fractions,


you will need to study the two examples above carefully and practice
with other examples

2.5.2 Significant Figures


Sometimes we are asked to express a number correct to a certain number
of decimal places or a certain number of significant figures.
Suppose that we wish to write the number 23.541638 correct to two
decimal places. To do this, we truncate the part of the number following
the second digit after the decimal point:

23.54 | 1638.

Then, since the first neglected digit, 1 in this case, lies between 0 and 4,
then the truncated number, 23.54, is the required answer.
If we wish to write the same number correct to three decimal places, the
truncated number is
23.541 | 638,
and since the first neglected digit, 6 in this case, lies between 5 and 9,
then the last digit in the truncated number is rounded up by 1.
Therefore, the number 23.541638 is 23.542 correct to three decimal
places or, for short, ‘to three decimal places’.

To express a number to a certain number of significant figures, we


employ the same rounding strategy used to express numbers to a certain
number of decimal places but we start counting from the first non-zero
digit rather than from the first digit after the decimal point.
For example,

72,648 = 70,000 (correct to 1 significant figure)


= 73,000 (correct to 2 significant figures)
= 72,600 (correct to 3 significant figures)

31
32 Unit 2: Equations and Inequalities

= 72,650 (correct to 4 significant figures),


and
0.004286 = 0.004 (correct to 1 significant figure)
= 0.0043 (correct to 2 significant figures)
= 0.00429 (correct to 3 significant figures).

Note that 497 = 500 correct to 1 significant figure and also correct to 2
significant figures.
Example

a) A teacher may ask for an exact answer to the problem “What


is the length of the diagonal of a square whose sides have
length 2?” The exact answer is √8. An approximate answer
is 2.8284.
b) A somewhat more complicated example is 3227/555 =
5.8144144144…, here the decimal representation becomes
periodic at the second digit after the decimal point, repeating
the sequence of digits "144" indefinitely.

Unit 2: Equations and Inequalities

2.1 Linear Equations


A mathematical statement setting two algebraic expressions equal to each
other is called an equation.
The simplest type of equation is the linear equation in a single variable
or unknown, which we will denote by x for the moment.
In a linear equation, the unknown x only occurs raised to the power 1.
The following are examples of linear equations:
i. 5𝑥 + 3 = 11,
ii. 1 − 4𝑥 = 3𝑥 + 7,
A linear equation may be solved by rearranging it so that all terms
involving x appear on one side of the equation and all the constant terms
appear on the other side.
Not all equations have solutions. In fact, equations may have no solutions
The solution of an at all or may have infinitely many solutions. Each of these situations is
equation is simply demonstrated in the following examples.
the value or values
of the unknown(s)
for which the left-
hand side (LHS) of
the equation is
equal to the right-
BBA 120 Business Mathematics

Case 1: Unique solutions An example of this is given above: 𝑥 + 4 =


10 etc.
Case 2: Infinitely many solutions The equation, 𝑥 + 𝑦 = 10 has
solutions (𝑥 = 5, 𝑦 = 5); (𝑥 = 4, 𝑦 = 6); (𝑥 = 3, 𝑦 = 7), etc. In
fact, this equation has infinitely many solutions or pairs of values (x, y)
which satisfy the formula, 𝑥 + 𝑦 = 10.
Case 3: No solution The equation, 0(𝑥) = 5 has no solution. There is
simply no value of x which can be multiplied by 0 to give 5.
Solving equations can involve a variety of techniques, many of which
will be covered later
Examples
(a) Given the equation 𝑥 + 3 = 2𝑥 − 6 + 5𝑥, solve for x.
Solution
𝑥 + 3 = 2𝑥 − 6 + 5𝑥
𝑥 + 3 = 7𝑥 − 6 (adding the x terms on the RHS)
𝑥 + 3 + 6 = 7𝑥 (bringing over –6 to the other side)
𝑥 + 9 = 7𝑥
9 = 7𝑥 − 𝑥 (bringing x over to the other side)
9 = 6𝑥
9
= 𝑥 (dividing both sides by 6)
6

1.5 = 𝑥
𝑥 𝑥
(b) Solve the equation − 3 = +1
4 5
Solution.
We go through the solution step-by-step. The idea is to rearrange
the equation so that all terms involving x appear on the left-hand
A common form of
side and all the constant terms appear on the right-hand side.
a linear equation in
Once this is done, the terms involving fractions are simplified.
the two variables x
𝑥
and y is 1. Subtract 5 from both sides:
𝒚 = 𝒎𝒙 + 𝒄 𝑥 𝑥
−5−3=1
4
where m and c 2. Add 3 to both sides
designate 𝑥 𝑥
constants. The 4
−5 =1+3=4
origin of the name 3. Simplify the left-hand side by expressing it as a single fraction.
"linear" comes This is achieved by expressing each of the fractions in terms of
from the fact that their lowest common denominator, 20. In the case of the first
the set of solutions fraction, both the numerator and denominator are multiplied by 5,
of such an equation
forms a straight
line in the plane. In
33
this particular
equation, the
constant m
34 Unit 2: Equations and Inequalities

and in the case of the second fraction they are both multiplied by
4, i.e.,
𝑥 5𝑥 5𝑥 𝑥 4𝑥 4𝑥
= = and = =
4 5×4 20 5 4×5 20

Therefore

5𝑥 4𝑥
= =4
20 20

5𝑥 − 4𝑥
=4
20

𝑥
=4
20

4. Finally multiply both sides by 20:

𝑥 = 80.

The solution to this equation is x = 80. Again we can check that this is the
correct solution by substituting x = 80 into the left- and right-hand sides
of our main equation.

2.2 Simple Inequalities


An equation is an equality. It states that the expression on the LHS of the
'=' sign is equal to the expression on the RHS.
An inequality is a relation that holds between two values when they are
different. In an inequality, expression on the LHS is either greater than
(denoted by the symbol >) or less than (denoted by the symbol <) the
expression on the RHS.
For example, 5 = 5 or 5𝑥 = 5𝑥 are equations, while
5 > 3 and 5𝑥 > 3𝑥 are inequalities which read, '5 is greater than 3';
'5x is greater than 3x' (for any positive value of x).

Inequality symbols
> greater than < less than
≥ greater than or equal to ≤ less than or equal to
BBA 120 Business Mathematics

2.2.1 Solving Inequalities


Solving linear inequalities is very similar to solving linear equations,
except for one small but important detail: you flip the inequality sign
whenever you multiply or divide the inequality by a negative.
When both sides of the inequality are multiplied or divided by negative
numbers or variables, then the statement remains true only when the
direction of the inequality changes: > becomes < and vice versa. For
example, multiply both sides of the inequality, 5 > 3 by -2:
5(−2) > 3(– 2) or – 10 > – 6 is not true

5(−2) < 3(−2), or −10 < −6 is true

We show this with some more examples:


Example 2.2.1
Find the range of values for which the following inequalities are true,
assuming that x > 0. State the solution in words and indicate the solution
on the number line.
75
a) 10 < 𝑥 − 12 b) − 𝑥
> 15 c) 2𝑥 − 6 ≤ 12 − 4𝑥
Solution
(a) 10 < 𝑥– 12 → 10 + 12 < 𝑥 → 22 < 𝑥 (𝑜𝑟 𝑥 > 22).
The solution states: 22 is less than x or .v is greater than 22. It is
represented by all points on the number line to the right of, but not
including, 22, as shown in the number line below

-10 0 10 22

75
(b) − 𝑥
> 15

Multiply both sides of the inequality by x. Since x > 0, the direction of


the inequality sign does not change.

−75 > 15𝑥

–5 > 𝑥 dividing both sides by 15

The solution states: -5 is greater than x or x is less than -5.

35
36 Unit 2: Equations and Inequalities

x cannot be less than -5 and greater than 0 at the same time. So there is no
solution

(c) 2𝑥 – 6 < 12 – 4𝑥

2𝑥 + 4𝑥 − 6 < 12 add 4x to both sides

6𝑥 < 12 + 6 add 6 to both sides

6𝑥 < 18

𝑥 < 3 dividing both sides by 6

 Solve x2 – 3x + 2 > 0

First, I have to find the x-intercepts of the associated quadratic, because


the intercepts are where y = x2 – 3x + 2 is equal to zero. Graphically, an
inequality like this is asking me to find where the graph is above or below
the x-axis. It is simplest to find where it actually crosses the x-axis, so I'll
start there.
Factoring, I get x2 – 3x + 2 = (x – 2)
(x – 1) = 0, so x = 1 or x = 2. Then the graph crosses the x-axis at 1
and 2, and the number line is divided into the intervals (negative infinity,
1), (1, 2), and (2, positive infinity). Between the x-intercepts, the graph is
either above the axis (and thus positive, or greater than zero), or else
below the axis (and thus negative, or less than zero).

There are two different algebraic ways of checking for this positivity or
negativity on the intervals. I'll show both.
1) Test-point method. The intervals between the x-intercepts are
(negative infinity, 1), (1, 2), and (2, positive infinity). I will pick a point
(any point) inside each interval. I will calculate the value of y at that point.
Whatever the sign on that value is, that is the sign for that entire interval.
For (negative infinity, 1), let's say I choose x = 0; then y = 0 – 0 + 2 = 2,
which is positive. This says that y is positive on the whole interval of
(negative infinity, 1), and this interval is thus part of the solution (since I'm
looking for a "greater than zero" solution).
For the interval (1, 2), I'll pick, say, x = 1.5; then y = (1.5)2 – 3(1.5) + 2
= 2.25 – 4.5 + 2 = 4.25 – 4.5 = –0.25, which is negative. Then y is
negative on this entire interval, and this interval is then not part of the
solution.
For the interval (2, positive infinity), I'll pick, say, x = 3; then y = (3)2 –
3(3) + 2 = 9 – 9 + 2 = 2, which is positive, and this interval is then part
of the solution. Then the complete solution for the inequality is x < 1 and
x > 2. This solution is stated variously as:
BBA 120 Business Mathematics

inequality notation

interval, or set, notation

number line with parentheses


(brackets are used
for closed intervals)

number line with open dots


(closed dots are used
for closed intervals)

The particular solution format you use will depend on your text, your
teacher, and your taste. Each format is equally valid. Copyright © Elizabeth
Stapel 1999-2011 All Rights Reserved

2) Factor method. Factoring, I get y = x2 – 3x + 2 = (x – 2)(x – 1). Now


I will consider each of these factors separately.
The factor x – 1 is positive for x > 1; similarly, x – 2 is positive for x > 2.
Thinking back to when I first learned about negative numbers, I know that
(plus)×(plus) = (plus), (minus)×(minus) = (plus), and (minus)×(plus) =
(minus). So, to compute the sign on y = x2 – 3x + 2, I only really need to
know the signs on the factors. Then I can apply what I know about
multiplying negatives.

First, I set up a grid,


showing the factors
and the number line.

Now I mark the


intervals where each
factor is positive.

Where the factors


aren't positive, they
must be negative.

Now I multiply up the


columns, to compute
the sign of y on each
interval.

Then the solution of x2 – 3x + 2 > 0 are the two intervals with the "plus"
signs:

37
38 Unit 2: Equations and Inequalities

(negative infinity, 1) and (2, positive infinity).

Rational Inequalities
 Solve x/(x – 3) < 2.

First off, I have to remember that I can't begin solving until I have
the inequality in "= 0" format.

Now I need to convert to a common denominator:

...and then I can simplify: Copyright © Elizabeth Stapel 1999-2011 All Rights
Reserved

The two factors are –x + 6 and x – 3. Note that x cannot equal 3,


or else I would be dividing by zero, which is not allowed. The first
factor, –x + 6, equals zero when x = 6. The other factor, x – 3,
equals zero when x = 3. Now, x cannot actually equal 3, so this
endpoint will not be included in any solution interval (even though
this is an "or equal to" inequality), but I need the value in order to
figure out what my intervals are. In this case, my intervals are
(negative infinity, 3), (3, 6], and [6, positive infinity). Note the use
of brackets to indicate that 6 can be included in the solution, but
that 3 cannot.

Using the Test-Point Method, I would pick a point in each interval


and test for the sign on the result. I could use, say, x = 0, x = 4,
and x = 7.

Using the Factor Method, I solve each factor: –x + 6 > 0 for –x >
–6, or x < 6; x – 3 > 0 for x > 3. Then I do the grid:
BBA 120 Business Mathematics

...fill in the signs on the factors:

...and solve for the sign on the rational function:

So the solution is all x's in the intervals (negative infinity, 3) and


[6, positive infinity).

2.3 Percentages
A percentage is a number or ratio as a
fraction of 100. It is often denoted using
the percent sign, “%”, or the abbreviation
“pct.”
Although percentages are usually used to
express numbers between zero and one,
any ratio can be expressed as a
percentage. For instance, 111% is 1.11
and −0.35% is −0.0035.

To convert a fraction to a percentage, we multiply the fraction by 100%.


For example,

3 3
= 4 × 100% = 75% ,
4
and
3 3
= 13 × 100% = 23.077% (to three decimal places).
13

39
40 Unit 2: Equations and Inequalities

To perform the reverse operation and convert a percentage to a fraction,


we divide the number by 100. The resulting fraction may then be
simplified to obtain a reduced fraction.

For example,

45 9
45% = 100 = 20 ,

where the fraction has been simplified by dividing the numerator and
denominator by 5 since this is a common factor of 45 and 100.

To find the percentage of a quantity, we multiply the quantity by the


number and divide by 100. For example,

25
25% of 140 is 100 × 140 = 35
and
4
4% of 5, 200 is 100 × 5,200 = 208

If a quantity is increased by a percentage, then that percentage of the


quantity is added to the original. Suppose that an investment of £300
increases in value by 20%. In monetary terms, the investment increases
by
20
× 300 = £60
100
and the new value of the investment is
£300 + £60 = £360.
In general, if the percentage increase is r%, then the new value of the
investment comprises the original and the increase. The new value can be
found by multiplying the original value by the factor

𝑟
1 + 100 .

It is easy to work in the reverse direction and determine the original value
if the new value and percentage increase is known. In this case, one
simply divides by the factor

𝑟
1 + 100 .

Examples
a) An investment rises from $2500 to $3375. Express the increase as a
percentage of the original.
Solution.
BBA 120 Business Mathematics

The rise in the value of the investment is


3375 − 2500 = 875
As a fraction of the original this is
875
= 0.35
2500
This is the same as 35 hundredths, so the percentage rise is 35%.

b) At the beginning of a year, the population of a small village is 8400. If


the annual rise in population is 12%, find the population at the end of the
year.
Solution
As a fraction
12
12% is the same as 100 = 0.12
so the rise in population is
0.12 × 8400 = 1008
Hence the final population is
8400 + 1008 = 9408
c) In a sale, all prices are reduced by 20%. Find the sale price of a good
originally costing $580.
Solution
As a fraction
20
20% is the same as = 0.2
100
so the fall in price is
0.2 × 580 = 116
Hence the final price is
580 − 116 = $464

d) The cost of a refrigerator is £350.15 including sales tax at 17.5%.


What is the price of the refrigerator without sales tax?

Solution. To determine the price of the refrigerator without sales tax, we


divide £350.15 by the factor
17.5
1+ = 1.175 .
100

So the price of the refrigerator without VAT is


350.15
= £298.
1.175

41
42 Unit 2: Equations and Inequalities

Similarly, if a quantity decreases by a certain percentage, then that


percentage of the original quantity is subtracted from the original to
obtain its new value. The new value may be determined by multiplying
the original value by the quantity
𝑟
1−
100

e) A person’s income is K25,000 of which K20,000 is taxable. If the rate of


income tax is 22%, calculate the person’s net income.

Solution. The person’s net income comprises the part of his salary that is
not taxable (K5,000) together with the portion of his taxable income that
remains after the tax has been taken. The person’s net income is therefore

22 78
5,000 + (1 − ) × 20000 = 5,000 + × 20,000
100 100

= 5,000 + 78 × 200

= 5,000 + 15,600

= 𝐾20,600

Exercises
1. Calculate

(a) 10% of $2.90 (b) 75% of $1250 (c) 24% of $580

2. A calculator has been marked up 15% and is being sold for


$78.50. How much did the store pay the manufacturer of the
calculator?
3. A shirt is on sale for $15.00 and has been marked down 35%. How
much was the shirt being sold for before the sale?
4. A firm’s annual sales rise from 50 000 to 55 000 from one year to the
next. Express the rise as a percentage of the original.
5. The government imposes a 15% tax on the price of a good. How much
does the consumer pay for a good priced by a firm at $1360?
6. Investments fall during the course of a year by 7%. Find the value of
an investment at the end of the year if it was worth $9500 at the
beginning of the year.
7. If the annual rate of inflation is 4%, find the price of a good at the end
of a year if its price at the beginning of the year is $25.
BBA 120 Business Mathematics

8. The cost of a good is $799 including 17.5% VAT (value added tax).
What is the cost excluding VAT?
9. Express the rise from 950 to 1007 as a percentage.
10. Current monthly output from a factory is 25 000. In a recession, this is
expected to fall by 65%. Estimate the new level of output.
11. As a result of a modernization programme, a firm is able to reduce the
size of its workforce by 24%. If it now employs 570 workers, how
many people did it employ before restructuring?
12. Shares originally worth $10.50 fall in a stock market crash to $2.10.
Find the percentage decrease.

2.4 Powers and Indices


Let 𝑥 be a number and 𝑛 be a positive
integer, then 𝑥 𝑛 denotes 𝑥 multiplied by
itself 𝑛 times. Here 𝑥 is known as the
base and 𝑛 is the power or index or
exponent. For example,
𝑥 5 = 𝑥 × 𝑥 × 𝑥 × 𝑥 × 𝑥.
More specifically, if n is a positive integer, we have
1 1
1. 𝑥 𝑛 = 𝑥 ⋅ 𝑥 ⋅ 𝑥 ⋅⋅⋅ 𝑥 3. 𝑥 −𝑛 = = for x≠ 0
𝑥𝑛 𝑥 ⋅ 𝑥 ⋅ 𝑥 ⋅⋅⋅ 𝑥
1
2. = 𝑥 𝑛 for x≠ 0 4. 𝑥 0 = 1
𝑥 −𝑛

If 𝑟 𝑛 = 𝑥, where n is a positive integer, then r is an nth root of x.


The Principal nth root of x is the nth root of x that is positive if x is
positive and negative if x is negative and n is odd.
𝑛
We denote the principle nth root of x by √𝑥

43
44 Unit 2: Equations and Inequalities

For example

2 3 3 1 1
√9 = 3, √−8 = −2, and √ =3
27

𝑛
The symbol √𝑥 is called a radical.
There are rules for multiplying and dividing two algebraic expressions or
numerical values involving the same base raised to a power.

The table below gives the Laws of Exponents.

𝑳𝒂𝒘 𝑬𝒙𝒂𝒎𝒑𝒍𝒆

1 𝑥1 = 𝑥 61 = 6

2 𝑥0 = 1 70 = 1

3 1 1
𝑥 −1 = 4−1 =
𝑥 4

4 𝑥 𝑚 𝑥 𝑛 = 𝑥 𝑚+𝑛 𝑥 2 𝑥 3 = 𝑥 2+3 = 𝑥 5

5 𝑥𝑚 𝑥6
= 𝑥 𝑚−𝑛 = 𝑥 6−2 = 𝑥 4
𝑥𝑛 𝑥2

6 (𝑥 𝑚 )𝑛 = 𝑥 𝑚𝑛 (𝑥 2 )3 = 𝑥 2×3 = 𝑥 6

7 (𝑥𝑦)𝑛 = 𝑥 𝑛 𝑦 𝑛 (𝑥𝑦)3 = 𝑥 3 𝑦 3
BBA 120 Business Mathematics

8 𝑥 𝑛 𝑥𝑛 𝑥 2 𝑥2
( ) = 𝑛 ( ) = 2
𝑦 𝑦 𝑦 𝑦

9 1 1
𝑥 −𝑛 = 𝑥 −3 =
𝑥𝑛 𝑥3

And the law about Fractional Exponents:

10

Example 2.1
Simplify the following using the rules of indices:
𝑥2 𝑥2𝑦3
1. 2.
𝑥 3⁄2 𝑥4𝑦

Write down the values of the following without using a calculator


3 3
a) 3-3 b) 164 c) 16−4
1 3

d) 27 3 e) 42 f) 190

Rationalizing the denominator of a fraction is a procedure in which a


fraction having a radical in its denominator is expressed as an equal
fraction without a radical in its denominator.
Example 2.2
Rationalizing the Denominator
2 2 √2
a. b. 6 c.
√5 √3 √3
Example 2.3
Exponents
𝑥 −2 𝑦3
a. Eliminate the negative exponents in for x ≠ 0, z ≠ 0.
𝑧 −2

𝑥 27
b. Simplify for x ≠ 0, z ≠ 0.
𝑥 3 𝑦5

45
46 Unit 2: Equations and Inequalities

c. Simplify (x5y8)5

d. Simplify (x5/9y4/3)18

5
𝑥 1/5 𝑦 6/5
e. Simplify ( )
𝑧 2/5

Example 2.4
Radicals
4
a. Simplify √48
b. Rewrite √2 + 5𝑥 without using a radical sign.
5
√2
c. Rationalize the denominator of 3 and simplify.
√6
√20
d. Simplify
√5

2.5 Quadratic Equations


The simplest non-linear equation is known as a quadratic equation and
takes the form
𝑓 (𝑥) = 𝑎𝑥 2 + 𝑏𝑥 + 𝑐
for some parameters a, b and c. In other words, a quadratic equation must
have a squared term as its highest power
For the moment we concentrate on the mathematics of quadratics and
show how to solve quadratic equations.
The values of x that satisfy the equation f(x) = 0 are known as the roots
or solutions of the equation. These two terms are used interchangeably.
Therefore, we say that 𝑥 = −2 and 𝑥 = 1/2 are the roots or solutions
of the quadratic equation 2𝑥 2 + 3𝑥 − 2 = 0.
The name Consider the elementary equation
Quadratic comes
𝑥2 − 9 = 0
from "quad"
It is easy to see that the expression on the left-hand side is a special case
meaning square,
of the above ( 𝑓 (𝑥) = 𝑎𝑥 2 + 𝑏𝑥 + 𝑐) with a = 1, b = 0 and c = −9. To
because the
solve this equation we add 9 to both sides to get
variable gets
squared (like x2). 𝑥2 = 9
BBA 120 Business Mathematics

so we need to find a number, x, which when multiplied by itself produces


the value 9.
A moment’s thought should convince you that there are exactly two
numbers that work, namely
3 and −3 because
3 × 3 = 9 and (−3) × (−3) = 9
These two solutions are called the square roots of 9. The symbol √ is
reserved for the positive square root, so in this notation the solutions are
√9 and −√9. These are usually combined and written±√9.

The equation
𝑥2 − 9 = 0
is trivial to solve because the number 9 has obvious square roots.

2.5.1 Solving Quadratic Equations


The solution of a quadratic equation is the value of x when you set the
equation equal to zero i.e. When you solve the following general
equation: 0 = 𝑎𝑥² + 𝑏𝑥 + 𝑐
There are a number of techniques for determining the roots of a quadratic
equation. Below are the four most commonly used methods to solve
quadratic equations.
 The Quadratic Formula
 Factoring
 Completing the Square
 Factor by Grouping

Solving by Factorization Method


If the expression defining a quadratic function can be factorized as a
product of linear factors, then equating each of the factors to zero and
solving the resulting linear equations will provide the roots.
Example 2.5.1
Solve 𝑥 2 + 13𝑥 + 30 = 0 using factorization.
Solution.
First, we factorize the quadratic expression 𝑥 2 + 13𝑥 + 30 as a product
of two linear factors (x + A) and (x + B), where A and B are two
constants that need to be determined. Since

47
48 Unit 2: Equations and Inequalities

(𝑥 + 𝐴)(𝑥 + 𝐵) = 𝑥 2 + (𝐴 + 𝐵)𝑥 + 𝐴𝐵,


then the constants A and B need to be chosen so that
A + B = 13, AB= 30.
The possible combinations of integers whose product is 30 are 30 × 1, 15
× 2, 10×3, and 6×5. Of course, one also has the combinations in which
the integers have been negated such as (−30)×(−1), but out of these
combinations the only one for which the pair of integers sums to 13 is 10
× 3. Therefore, we choose
A = 10 and B = 3, i.e.,
𝑥 2 + 13𝑥 + 10 = (𝑥 + 10)(𝑥 + 3).
We now solve the equation (x+3)(x + 10) = 0.
For the product of the two linear terms x+3 and x+10 to be zero, at least
one of them must be zero. So either
𝑥 + 3 = 0 𝑜𝑟 𝑥 + 10 = 0.
If x + 3 = 0 then x = −3, and if x + 10 = 0 then x = −10.
Therefore, the roots of the equation x2 + 13x + 30 = 0 are x = −3 and x =
−10.

Example 2
Solve the quadratic equation 2𝑥 2 − 11𝑥 + 12 = 0 using factorization.
Solution.
As in the previous example, the first step is to factorize the quadratic
expression 2𝑥 2 − 11𝑥 + 12 as a product of linear factors.
These linear factors must be of the form (2x + A) and (x + B) in order to
retrieve the quadratic factor 2x2, where A and B are two positive
constants.
Since
(2𝑥 + 𝐴)(𝑥 + 𝐵) = 2𝑥 2 + (𝐴 + 2𝐵)𝑥 + 𝐴𝐵,
then the constants A and B need to be chosen so that
A + 2B = −11,
AB= 12.
The possible combinations of integers whose product is 12 are 12 × 1, 6 ×
2, 4 × 3, −4× −3, −6× −2, and −12 × −1.
The only pair of integers amongst these for which A + 2B = −11 is A =
−3 and B = −4. Therefore, we have
2𝑥 2 − 11𝑥 + 12 = (2𝑥 − 3)(𝑥 − 4).
The problem now is to solve the equation
(2𝑥 − 3)(𝑥 − 4) = 0.
BBA 120 Business Mathematics

Either 2x − 3 = 0 or x − 4 = 0. If 2x − 3 = 0 then 2x = 3 and x = 3/2.


If x−4 = 0, then x = 4.
Therefore, the two roots of the equation 2𝑥 2 − 11𝑥 + 12 = 0 are x =
3/2 and x = 4.

Most quadratic expressions, however, do not factorize easily in the sense


that they cannot be expressed as a product of linear factors with integer
coefficients, even if the coefficients of the quadratic equation are
integers.
For example, the quadratic equation 3𝑥 2 − 9𝑥 + 5 = 0 cannot be
factored into a product of linear factors with integer coefficients. Clearly,
a more systematic approach is required.

The Quadratic Formula


A quadratic equation has the general form
𝑎𝑥 2 + 𝑏𝑥 + 𝑐 = 0
where a, b and c are constants.
The solution of this equation may be found by using the formula,
−𝑏 ± √𝑏 2 − 4𝑎𝑐
𝑥=
2𝑎
The following examples describes how to use this formula. It also
illustrates the fact that a quadratic equation can have two solutions, one
solution or no solutions.
Example 2.5.2
Solve the quadratic equations
(a) 2𝑥 2 + 9𝑥 + 5 = 0
(b) 𝑥 2 − 4𝑥 + 4 = 0
(c) 3𝑥 2 − 5𝑥 + 6 = 0
Solution
(a) For the equation
2x2 + 9x + 5 = 0
we have a = 2, b = 9 and c = 5. Substituting these values into the formula
−𝑏 ± √𝑏 2 − 4𝑎𝑐
𝑥=
2𝑎

Gives

49
50 Unit 2: Equations and Inequalities

−9 ± √92 − 4(2)(5)
𝑥=
2(2)
−9 ± √81 − 40
𝑥=
4
−9 ± √41
𝑥=
4
The two solutions are obtained by taking the + and − signs separately:
that is,
−9 + √41
𝑥= = −0.649
4

−9 − √41
𝑥= = −3.851
4
It is easy to check that these are solutions by substituting them into the
original equation. For example, putting x = −0.649 into
2𝑥 2 + 9𝑥 + 5
gives
2(−0.649)2 + 9(−0.649) + 5 = 0.001 402
which is close to zero, as required. We cannot expect to produce an exact
value of zero because we rounded √41 to 3 decimal places. You might
like to check for yourself that −3.851 is also a solution.
(b) For the equation
𝑥 2 − 4𝑥 + 4 = 0
we have a = 1, b = −4 and c = 4. Substituting these values into the
formula
−𝑏 ± √𝑏 2 − 4𝑎𝑐
𝑥=
2𝑎
Gives
−(−4) ± √(−4)2 − 4(1)(4)
𝑥=
2(1)
4 ± √16 − 16
=
2
4 ± √0
=
2
4 ± √0
=
2
Clearly we get the same answer irrespective of whether we take the + or
the − sign here. In other words, this equation has only one solution, x = 2.
As a check, substitution of x = 2 into the original equation gives
(2)2 − 4(2) + 4 = 0
BBA 120 Business Mathematics

c) For the equation


3𝑥 2 − 5𝑥 + 6 = 0
we have a = 3, b = −5 and c = 6. Substituting these values into the
quadratic formula gives
−(−5) ± √(−5)2 − 4(3)(6)
𝑥=
2(3)
5 ± √25 − 72
=
6
5 ± √(−47)
=
6
The number under the square root sign is negative and it is impossible to
find the square root of a negative number. We conclude that the quadratic
equation
3𝑥 2 − 5𝑥 + 6 = 0
has no real solutions.

Practice Problems
Solve the following quadratic equations (where possible):
(a) 2𝑥 2 − 19𝑥 − 10 = 0 (b) 4𝑥 2 + 12𝑥 + 9 = 0
(c) 𝑥 2 + 𝑥 + 1 = 0 (d) 𝑥 2 − 3𝑥 + 10 = 2𝑥 + 4

51
52 Unit 2:Equations and Inequalities

Unit 2:Equations and Inequalities


Example
Solve the quadratic equation 3𝑥 2 − 9𝑥 + 5 = 0 using the formula.
Solution.
First we compare the coefficients of this equation with those of the
general quadratic equation.
If we do this, we notice that 𝑎 = 3, 𝑏 = −9, and 𝑐 = 5.
Inserting these values into the quadratic formula gives
−(−9) ± √(−9)2 − 4 × 3 × 5
𝑥=
2×3
9 ± √81 − 60
=
6
9 ± √21
=
6
Note that 21 is not a perfect square, and therefore the roots of this
equation can only be expressed in decimal representation to a specified
number of decimal places. Therefore, to four decimal places the two
solutions of this equation are
9+√21 9−√21
x= 6
= 2.2638 and, x= 6
= 0.7362

Completing the Square Method


Some quadratics cannot be easily factorized so we have to use the
technique of "Completing the Square."
The Idea behind completing the square is to rearrange the quadratic into
the neat "(squared part) equals (a number)" format. i.e (𝑥 − 4)2 = 23.
We show this method using an example.

This is the original problem. 4x2 – 2x – 5 = 0

Move the loose number over to the other


4x2 – 2x = 5
side.
BBA 120 Business Mathematics

Divide through by whatever is multiplied


on the squared term.
Take half of the coefficient (don't forget
the sign!) of the x-term, and square it. Add
this square to both sides of the equation.
Convert the left-hand side to squared
form, and simplify the right-hand side.
(This is where you use that sign that you
kept track of earlier. You plug it into the
middle of the parenthetical part.)

Square-root both sides, remembering the


"±" on the right-hand side. Simplify as
necessary.

Solve for "x =".

Remember that the "±" means that you


have two values for x.

Solve x2 + 6x – 7 = 0 by completing the square.


Do the same procedure as above, in exactly the same order.

This is the original equation. x2 + 6x – 7 = 0

Move the loose number over to the other side. x2 + 6x =7

Take half of the x-term (that is, divide it by two) (and


don't forget the sign!), and square it. Add this square to x2 + 6x +9 = 7+9
both sides of the equation.

Convert the left-hand side to squared form. Simplify


(x + 3)2 = 16
the right-hand side.

Square-root both sides. Remember to do "±" on the


x+3=±4
right-hand side.

53
54 Unit 2:Equations and Inequalities

x=–3±4
Solve for "x =". Remember that the "±" gives you two
= – 3 – 4, –3 + 4
solutions. Simplify as necessary.
= –7, +1
Absolute-value
equations
always work this
way: to be able
to remove the
Solutions of a Quadratic Equation
absolute-value
bars, you have The number of solutions of a quadratic equation depends on the sign of
to isolate the the expression under the square root sign in this formula. A quadratic
absolute value equation has two, one or no solutions depending on whether the
onto one side, expression 𝑏 2 − 4𝑎𝑐 is positive, zero, or negative:
and then split
the equation  If 𝑏 2 − 4𝑎𝑐 > 0, there are two solutions
into the two −𝑏+√𝑏2 −4𝑎𝑐 −𝑏−√𝑏2 −4𝑎𝑐
𝑥= 2𝑎
and 𝑥= 2𝑎
.
possible cases.
 If 𝑏 2 − 4𝑎𝑐 = 0, then there is one solution
𝑏
𝑥 = − 2𝑎

 If 𝑏 2 − 4𝑎𝑐 < 0, then there are no solutions since the square root of
𝑏 2 − 4𝑎𝑐 does not exist in this case.

2.6 Absolute Value


For any real number x the absolute value or modulus of x is denoted by
| x | and is defined as

As can be seen from the above definition, the absolute value of x is


always either positive or zero, but never negative.
Since the square-root notation without sign represents the positive square
root, it follows that

Solving Absolute Value Equations


The absolute value property (that both the positive and the negative
become positive) makes solving absolute-value equations a little tricky.
But once you learn the "trick", they're not so bad. Let's start with
something simple:
 Solve | x | = 3
BBA 120 Business Mathematics

From the definition of a modulus, | 3 | = 3 and | –3 | = 3, so x must be 3 or


–3.
Then the solution is x = –3, 3.

 Solve | x + 2 | = 7
To clear the absolute-value bars, we must split the equation into its two
possible two cases, one case for each sign:
(x + 2) = 7 or –(x + 2) = 7
x+2=7 or –x – 2 = 7
x=5 or –9 = x
Then the solution is x = –9, 5.

 Solve | x2 – 4x – 5 | = 7
First, I'll clear the absolute-value bars by splitting the equation into its
two cases:
( x2 – 4x – 5 ) = 7 or –(x2 – 4x – 5) = 7
Solving the first case, we get:
x2 – 4x – 5 = 7
x2 – 4x – 12 = 0
(x – 6)(x + 2) = 0
x = 6, x = –2
Solving the second case, we get:
–x2 + 4x + 5 = 7
–x2 + 4x – 2 = 0
0 = x2 – 4x + 2
Applying the Quadratic Formula to the above, we get:

Then my solution is:


𝑥 = −2, 6, 2 ± √2

Practice Problems
Solve a) |2𝑥 + 1| = 3
b) |𝑥| < |𝑥 + 4|

55
56 Unit 3: Functions and Graphs

Unit 3: Functions and Graphs

3.0 Functions

Given two arbitrary sets X and Y, a function from X to Y is a rule which


assigns one member of Y to each member of X.

Notation
A function f with domain X and codomain Y is commonly denoted by
𝑓: 𝑋 → 𝑌
or
The above diagram 𝑓
represents a function 𝑋→𝑌
with domain {1, 2, 3},
codomain {A, B, C, D}. In our function above, the elements of X are called arguments of f.
 the set "X" is called the Domain,
 the set "Y" is called the Codomain, and
 the set of elements that get pointed to in Y (the actual values
produced by the function) is called the Range.
For each argument x, the corresponding unique y in the codomain is
called the function value at x or the image of x under f. It is written as
f(x).
However this second One says that f associates y with x or maps x to y. This is abbreviated by
diagram does not
represent a function 𝑦 = 𝑓(𝑥)
since 2 is the first Example
element in more than
one ordered pair. If X and Y are both the set ℝ, the rule which says ‘multiply by 2’ is a
function.
Normally we express this function by a formula. If we call the function f
we can write the rule which define 𝑓(𝑥) = 2𝑥.
It is worth noting that when we can define f in this way the 𝑥 is a ‘dummy
variables, the function could equally well be defined by writing
𝑓(𝑦) = 2𝑦, or 𝑓(𝑡) = 2𝑡 or even 𝑓(𝑏𝑙𝑜𝑏) = 2𝑏𝑙𝑜𝑏.
The point is that f is the function which doubles any value.
The symbol used to denote a typical real number in the domain of a
function is called the independent variable. The symbol used to denote
the typical real number in the range is called the dependent variable.
It is often helpful to think of a function as a ‘black box’ which converts
an input into output (Figure 3.1). If the name of the function is 𝑓 then
𝑓(𝑥) is the output corresponding to a given input 𝑥.
BBA 120 Business Mathematics

The box may represent any rule such as ‘multiply by 2’, or ‘add 23’, or
‘square’, provided. Only that for each input there is a unique, well-
defined output.

𝑥 𝑓 𝑓(𝑥)

Figure 3.1: Diagrammatic representation of a function.

The diagram is useful because it stresses that a function is a one-way


relationship, signified by the direction of the arrows.
Sometimes it may not be possible to reverse the arrows that is, the output
may not determine the input. For example, if the function if the ‘square’
function, so that 𝑓(𝑥) = 𝑥 2 , and the ‘output’ of the box is 4, we do not
know whether the input was 2 or 2, either of which would give the same
value.

3.1 Inverse Functions


If 𝑓is a function for which it is possible to reverse the arrows, the
resulting ‘reverse’ function is called the Inverse function for 𝑓 and is
denoted by 𝑓 −1 so if, then 𝑥 = 𝑓 −1 (𝑦)

𝑥 𝑓 𝑦 𝑦 𝑓 −1 𝑥

Figure 3.2. A function and its inverse.


Example

Suppose f is given by the formula 𝑓(𝑥) = 2𝑥 + 7 so that when the input


is x the output is 𝑦 = 2𝑥 + 7.
To obtain a formula for the inverse function we have to find out what
value of x will give a particular y, and we can do this simply by
7
reorganizing the equation 𝑦 = 2𝑥 + 7 into the form 𝑥 = (𝑦 − 2).

This uniquely determines x in terms of y, so here we have the inverse


function 𝑓 −1 given by the formula.

7
𝑓 −1 (𝑦) = (𝑦 − 2)

57
58 Unit 3: Functions and Graphs

Note that if f and g are functions such that g is the inverse of f, then f is
the inverse of g.
For 𝑦 = 𝑓(𝑥) if and only if 𝑥 = 𝑔(𝑦) by the definition of 𝑔 = 𝑓 −1 .
Here 𝑥 and 𝑦 are any values, and their names are irrelevant.
So we may interchange the names, giving 𝑦 = 𝑔(𝑥) if and only if 𝑥 =
𝑓(𝑦) and this is just the condition that 𝑓 = 𝑔−1 .

If 𝑓 has an inverse function 𝑓 −1 , one can think of 𝑓 −1 as the ’undoing’


of 𝑓 in the sense that if we take 𝑥 and form 𝑦 = 𝑓(𝑥) and then apply 𝑓 −1
to, 𝑦 we obtain 𝑥 again.
For example, consider again the previous example. Here
7
f ( x)  2 x  7 and f 1 ( y)  ( y  ).
2
For any
(2𝑥 + 7) − 7 2𝑥
𝑓 −1 (𝑓(𝑥)) = 𝑓 −1 (2𝑥 + 7) = = =𝑥
2 2

Thus, for any 𝑥,


𝑓 −1 (𝑓(𝑥)) = 𝑥 ,

The observation just made concerns following the action of 𝑓 by that of


𝑓 −1 .
The notion of apply one function directly to the output of another is an
important one and can be made precise, as follows.

Practice Problems

1. Given 𝑓(𝑥) = 3𝑥 − 2 find 𝑓 −1 (𝑥)

2. Given 𝑔(𝑥) = √𝑥 − 3 find , 𝑔−1 (𝑥)

𝑥+4
3. Given ℎ(𝑥) = 2𝑥−5 find ℎ−1 (𝑥).

3.2 Composite Functions.


Composition functions are used to combine the two functions. If A, B and
C be any three sets and 𝑓: 𝐴 → 𝐵 and 𝑔: 𝐵 → 𝐶 be any two functions,
then the domain of g is the co-domain of 𝑓.
The composition of functions f and g is denoted by "𝑔 𝑜 𝑓" or we can say
that the function g o f is called the composition of two functions f and g.
BBA 120 Business Mathematics

Definition
Let 𝑓: 𝐴 → 𝐵 and 𝑔: 𝐵 → 𝐶 be two functions. Then, the
composition of functions f and g denoted by "g o f".
𝑔 o 𝑓 is read as 𝑔 of 𝑓.
The new function (𝑔 𝑜 𝑓): 𝐴 → 𝐶 such that (𝑔 𝑜 𝑓) (𝑎) =
𝑔(𝑓(𝑎)) for all 𝑎 ∈ 𝐴. Here, 𝑓(𝑎) is an element of B. Therefore,
𝑔(𝑓(𝑎)) is meaningful.

The function 𝑔 𝑜 𝑓: 𝐴 → 𝐶 given by 𝑔 𝑜 𝑓(𝑎) = 𝑔(𝑓(𝑎)) for all


"a" belongs to A.
This is illustrated in the diagram below

Figure 3.3: A composite Function.

A composite function is sometimes called ‘function of a function’.

Given three functions 𝑓, 𝑔, ℎ, the composite 𝑓 ∘ 𝑔 ∘ ℎ, is defined to be


𝑓(𝑔 ∘ ℎ). In other words, we first form ℎ followed by 𝑔 and then form
the composite of this function (𝑔 ∘ ℎ). followed by 𝑓.
Note that composition of a function is an associative operation.

Examples:
1. Let A = {1, 2}, B = {3, 4} and C = {5, 6} and 𝑓: 𝐴 → 𝐵 and 𝑔: 𝐵 → 𝐶
such that f(1) = 3, f(2) = 4, g(3) = 5, g(4) = 6. Find g o f.

Solution:

59
60 Unit 3: Functions and Graphs

g o f is a function from A → C. Identify the images of elements of A


under the function g o f.

Domain for g = {f(1), f(2)}


(g o f) (1) = g(f(1)) = g(3) = 5
(g o f) (2) = g(f(2)) = g(4) = 6
i.e. image of 1 is 5 and image of 2 is 6 under g o f. That is, g o f = {(1, 5),
(2, 6)}.

2. The two functions f, g are defined by 𝑓(𝑥) = 𝑥 2 + 1, 𝑔(𝑥) = 𝑥 − 1.


Find 𝑓 𝑜 𝑔 and 𝑔 𝑜 𝑓 and show that 𝑓 𝑜 𝑔 ≠ 𝑔 𝑜 𝑓.

Solution:
Given functions are f(x) = x2 + 1, g(x) = x - 1
(f o g) (x) = f(g(x)) = f(x - 1) = (x - 1)2 + 1 = x2 - 2x + 2
(g o f) (x) = g(f(x)) = g(x2 + 1) = (x2 + 1) - 1 = x2
Thus, (f o g) (x) = x2 - 2x + 2
(g o f) (x) = x2

fog≠gof

3. The two function f and g are f(x) = 3x, g(x) = x + 1. Find g o f.

Solution:
Given f(x) = 3x, g(x) = x + 1
g o f = g(f(x) = g(3x) = 3x + 1
g o f = 3x + 1

Example

Suppose that
BBA 120 Business Mathematics

3
1
f ( x)  , g ( x)  x 2 and h( x)  x 2  2 x  3. Then
x
3 3

( g  h)( x)  g ( x 2  2 x  3)  ( x 2  2 x  3) 2 and ( f  g  h)( x)  f ( x 2  2 x  3) 2 )


1
 3

( x 2  2 x  3) 2

3.3 Special Functions


3.3.1 Constant Functions

Let ℎ(𝑥) = 2. The domain of ℎ is all real numbers. All function values
are 2. For example ℎ(10) = 2, ℎ(−328) = 2, ℎ(𝑥 + 3) = 2
We call ℎ a constant function because all the function values are the
same. More generally, we have this definition.

A function of the form ℎ(𝑥) = 𝑐 where C is a constant function is called


a constant function.

3.3.2 Polynomial Functions


A constant function belongs to a broader class of functions, called
polynomial functions.
In general, a function of the form

𝑓(𝑥) = 𝐶𝑛 𝑥 𝑛 + 𝐶𝑛−1 𝑥 𝑛−1 + ⋯ + 𝐶1 𝑥 + 𝐶0 ,


where n is a nonnegative integer and 𝐶𝑛 , 𝐶𝑛−1 … 𝐶0 are constants with
𝐶𝑛 ≠ 0 is called a polynomial function (in 𝑥).
The number 𝑛 is called the degree of polynomial, and 𝐶𝑛 is the leading
coefficient.
Thus 𝑓(𝑥) = 3𝑥 2 − 8𝑥 + 9 is a polynomial function of degree 2 with
leading coefficient 3.
Likewise 𝑔(𝑥) = 4 − 2𝑥 has degree 1 and leading coefficient 2.
Polynomial functions of degree 1 or 2 are called linear or quadratic
functions, respectively. Hence 𝑔(𝑥) = 4 − 2𝑥 is linear and 𝑓(𝑥) =
3𝑥 2 − 8𝑥 + 9 is quadratic.
Note that a nonzero constant function such as 𝑓(𝑥) = 5 [which can be
written as (𝑥) = 5𝑥 0 ], is a polynomial function of degree 0.

61
62 Unit 3: Functions and Graphs

Examples
a. 𝑓(𝑥) = 𝑥 3 − 6𝑥 2 + 7 is a polynomial (function) of degree 3 with
leading coefficient 1.
2𝑥 2
b. 𝑔(𝑥) = 3
is a linear function with leading coefficient 3.
2
c. 𝑓(𝑥) = 3 is not a polynomial function, because 𝑓(𝑥) = 2𝑥 −3
𝑥
and the exponent for 𝑥 is not a nonnegative integer.
1
Similarly, 𝑔(𝑥) = √𝑥 is not a polynomial because 𝑔(𝑥) = 𝑥 2

3.3.3 Rational Functions


𝑎
A function written in the form of 𝑏 is called rational function, 𝑓(𝑥) =
𝑝(𝑥)
𝑞(𝑥)
, where 𝑝(𝑥) and 𝑞(𝑥) are polynomial functions.

Example
𝑥 2 −6𝑥
a) 𝑓(𝑥) = 𝑥+5
Note that this rational function is not defined for
𝑥 = −5 , since division by 0 is not defined.

2𝑥+3
b) 𝑔(𝑥) = 2𝑥 + 3 is a rational function since = 2𝑥 + 3
1

In fact, every polynomial function is also a rational function.

3.3.4 Compound Functions

1, if  1  s  1
Let f ( s)  0 If 1  s  2
s  3, if 2  s  8

This is called a compound function because the rule for this function is
given by more than one expression.
Here 𝑠 is the independent variable and the domain 𝑓 is all 𝑠 such that
−1 ≤ 𝑠 ≤ 8

Example
Find 𝑓(0), 𝑓(2) and 𝑓(7)
Answers
BBA 120 Business Mathematics

𝑓(0) and 𝑓(2) = 0, 𝑓(7) = 4

3.3.5 Absolute Value Function

The function 𝑓(𝑥) = |𝑥| is called the absolute value function.


Recall that the absolute value or magnitude, if a real number 𝑥 is denoted
|𝑥| and is defined by

x, if x  0
x 
 x, if x  0
Thus the domain of 𝑓 is all real numbers.
Some function values are:
f (16)  16  16
  4  4   4 4
f    
 3  3  3  3
f (0)  0  0
Example
Solve |𝑥 − 3| = 2 solving this gives 𝑥 = 5 𝑜𝑟 1

Unit 3: Functions and Graphs

3.4 One to One, Many to One and


Onto Functions
Recall that a function is a set of ordered pairs in which no two
ordered pairs that have the same first component have different
second components.
This means that given any x, there is only one y that can be paired
with that x.

3.4.1 Onto Function


A function f from A to B is called onto if for all b in B there is an a in A
such that f (a) = b. All elements in B are used.

63
64 Unit 3: Functions and Graphs

Such functions are referred to as surjective.

"Onto" NOT "Onto"


(all elements in B are used) (the 8 and 1 in Set B are not used)

By definition, to determine if a function is ONTO, you need to


know information about both set A and B.
When working in the coordinate plane, the sets A and B may both
become the Real numbers, stated as 𝑓: ℝ → ℝ.

EXAMPLE 1: Is 𝑓 (𝑥) = 3𝑥 − 4 onto where : ℝ → ℝ ?

This function (a straight line) is


ONTO.

As you progress along the line,


every possible y-value is used.

In addition, this straight line also possesses the


property that each x-value has one unique y-
value that is not used by any other x-
element. This characteristic is referred to as
being one-to-one.

EXAMPLE 2: Is 𝑔 (𝑥) = 𝑥² − 2 onto where 𝑔: ℝ → ℝ?


BBA 120 Business Mathematics

This function (a parabola) is


NOT ONTO.

Values less than -2 on the y-axis are never


used. Since possible y-values belong to the set
of ALL Real numbers, not ALL possible y-
values are used.

In addition, this parabola also has y-values that


are paired with more than one x-value, such as
(3, 7) and (-3, 7).
This function will not be one-to-one.

EXAMPLE 3: Is 𝑔 (𝑥) = 𝑥² − 2 onto where 𝑔: ℝ → [−2, ∞)?


If set B is redefined to be [−2, ∞) , ALL of the possible y-values are now used, and function g
(x) (under these conditions) is ONTO.

3.4.2 One to One Functions (Injections)


A function f from A to B is called one-to-one (or 1-1) if whenever
f (a) = f (b) then a = b. No element of B is the image of more
than one element in A.

In a one-to-one function, given any y there is only one x that can be


paired with the given y. Such functions are referred to as injective.

65
66 Unit 3: Functions and Graphs

"One-to-One" NOT "One-to-One"


BBA 120 Business Mathematics

EXAMPLE 1: Is f (x) = x³ one-to-one where 𝑓: ℝ → ℝ ?

This function is One-to-One.

This cubic function possesses the property that


each x-value has one unique y-value that is not used
by any other x-element. This characteristic is
referred to as being 1-1.

Also, in this function, as you progress along the


graph, every possible y-value is used, making the
function onto.

EXAMPLE 2: Is 𝑔 (𝑥) = | 𝑥 − 2 | one-to-one where 𝑔: ℝ → [0, ∞)? ?

This function is
NOT One-to-One.

This absolute value function has y-values that are


paired with more than one x-value, such as (4, 2)
and (0, 2).
This function is not one-to-one.

In addition, values less than 0 on the y-axis are


never used, making the function NOT onto.

67
68 Unit 3: Functions and Graphs

3.4.2 One-to-One and Onto (Bijections)

A Bijection is a function which is both a surjection and an injection, or


using other words, a bijection is a function which is both "one-to-one"
and "onto".
Examples
1. The function 𝑓: 𝑹 → 𝑹, 𝑓(𝑥) = 2𝑥 + 1 is bijective, since for each y
𝑦−1
there is a unique 𝑥 = such that 𝑓(𝑥) = 𝑦.
2
In more generality, any linear function over the reals, f: R → R, 𝑓(𝑥) =
𝑎𝑥 + 𝑏 (where a is non-zero) is a bijection. Each real number y is
𝑦–𝑏
obtained from (paired with) the real number 𝑥 = 𝑎
.

2. The exponential function, g: R → R, 𝑔(𝑥) = 𝑒 𝑥 , is not bijective: for


instance, there is no x in R such that g(x) = −1, showing that g is not onto
(surjective). However if the codomain is restricted to the positive real
numbers , then g becomes bijective; its inverse (see below)
is the natural logarithm function ln.

3. The function h: R → R+, h(x) = x2 is not bijective: for instance, h(−1) =


h(1) = 1, showing that h is not one-to-one (injective). However, if the
domain is restricted to , then h becomes bijective; its inverse
is the positive square root function.
BBA 120 Business Mathematics

Unit 4: Types of Functions

4.1 The Straight Line and its


Applications
4.1.1 Graphs of Linear Equations
Consider the two straight lines shown in Figure 1.1. The horizontal line is
referred to as the x axis and the vertical line is referred to as the y axis.
The point where these lines intersect is known as the origin and is
denoted by the letter O. These lines enable us to identify uniquely any
point, P, in terms of its coordinates (x, y).
The first number, x, denotes the horizontal distance along the x axis and
the second number, y, denotes the vertical distance along the y axis. The
arrows on the axes indicate the positive direction in each case.

Figure 1.1

Example
Plot the points A(2, 3), B(−1, 4), C(−3, −1), D(3, −2) and E(5, 0).
Solution
The point A with coordinates (2, 3) is obtained by starting at the origin,
moving 2 units to the right and then moving 3 units vertically upwards.
Similarly, the point B with coordinates (−1, 4) is located 1 unit to the left
of O (because the x coordinate is negative) and 4 units up. These points,
together with C(−3, −1), D(3, −2) and E(5, 0) are plotted in Figure 1.2.

69
70 Unit 4: Types of Functions

Note that the point C lies in the bottom left-hand quadrant since its x and
y coordinates are both negative.
It is also worth noticing that E actually lies on the x axis since its y
coordinate is zero. Likewise, a point with coordinates of the form (0, y)
for some number y would lie somewhere on the y axis. Of course, the
point with coordinates (0, 0) is the origin, O.

figure 1.2
Example
Decide which of the following points lie on the line 5x − 2y = 6:
A(0, −3), B(2, 2), C(−10, −28) and D(4, 8)
Solution
5(0) − 2(−3) = 0 − (−6) = 0 + 6 = 6
5(2) − 2(2) = 10 − 4 = 6
5(−10) − 2(−28) = −50 − (−56) = −50 + 56 = 6
5(4) − 2(8) = 20 − 16 = 4 ≠ 6
Hence points A, B and C lie on the line, but D does not lie on the line.

Example
Sketch the line
2x + y = 5
Solution
Setting x = 0 gives
2(0) + y = 5
0+y=5
y=5
BBA 120 Business Mathematics

Hence (0, 5) lies on the line.


Setting y = 0 gives
2x + 0 = 5
2x = 5
x = 5/2 (divide both sides by 2)
Hence (5/2, 0) lies on the line.
The line 2x + y = 5 is sketched in Figure 1.4. Notice how easy the algebra
is using this approach. The two points themselves are also slightly more
meaningful. They are the points where the line intersects the coordinate
axes.

Figure1.4

4.1.2 The Straight Line: slope, intercept and graph


The straight line is one of the simplest mathematical functions: it is easy
to manipulate and to graph. It is, therefore, essential to fully understand
the straight line, what each part of the equation means and the effect of
changes in each part on the graph of the line.

4.1.2.1 The equation of a line


The equation of a line is simply a formula which shows how the y-
coordinate is related to the x-coordinate for every point on the line.
The equation of a straight line has the general form
𝑦 = 𝑚𝑥 + 𝑐
where m is the symbol representing slope and c is the symbol
representing the intercept on the y-axis.

71
72 Unit 4: Types of Functions

4.1.2.2 How to define a straight line


A straight line may be defined by two properties:
• Slope, usually represented by the symbol m
• Vertical intercept, the point at which the line crosses the y-axis,
usually represented by the symbol c

4.1.2.3 Slope
The slope of a line is simply the 'slant' of the line. The slope is negative
if the line is falling from left to right and the slope is positive if the line is
rising from left to right.
Note the horizontal line has a zero slope and the vertical line has an
infinite slope.
Measuring the slope of a line
 The line through the points A and B
 has a positive slope since it is rising from left to right.
The slope is measured as follows:
𝑐ℎ𝑎𝑛𝑔𝑒 𝑖𝑛 ℎ𝑒𝑖𝑔ℎ𝑡 2 1
𝑆𝑙𝑜𝑝𝑒 = = = = 0.5
𝑐ℎ𝑎𝑛𝑔𝑒 𝑖𝑛 𝑑𝑖𝑠𝑡𝑎𝑛𝑐𝑒 4 2
That is, height increases by 0.5 units when the horizontal
distance increases by 1 unit.

 The line through the points C and D has a negative slope


since the line is falling from left to right. A decrease in
height is indicated by a minus sign. The slope is
measured as follows:
𝑐ℎ𝑎𝑛𝑔𝑒 𝑖𝑛 ℎ𝑒𝑖𝑔ℎ𝑡 −6
𝑆𝑙𝑜𝑝𝑒 = = = −1.2
𝑐ℎ𝑎𝑛𝑔𝑒 𝑖𝑛 𝑑𝑖𝑠𝑡𝑎𝑛𝑐𝑒 5
That is, height decreases by 1.2 units when the horizontal
distance increases 1 unit.

Note: The change in height and change in distance are generally


referred to as 𝛥𝑦 and 𝛥𝑥 respectively where the symbol 𝛥 means
change.

The slope of a line may be described in several ways:


1. The change in height (Δy) divided by the corresponding increase in
horizontal distance (Δx):
Δ𝑦
slope = Δ𝑥
2. The change in height per unit increase in horizontal distance
BBA 120 Business Mathematics

3. The number of units by which y changes when x increases by 1 unit


• Lines with the same slope, but different intercepts are different lines:
they are parallel
• Lines with the same intercept, but different slopes are different lines
• Two lines are identical only if their slopes and intercepts are identical

4.1.2.4 Calculating the slope of a line given two points on


the line
If two points on a line are known, then the slope is defined as the change
in the v-coordinate divided by the change in the .x-coordinate given as
𝐶ℎ𝑎𝑛𝑔𝑒 𝑖𝑛 𝑦 − 𝑐𝑜𝑜𝑟𝑑𝑖𝑛𝑎𝑡𝑒 Δ𝑦 𝑦2 − 𝑦1
𝑆𝑙𝑜𝑝𝑒 = = =
𝐶ℎ𝑎𝑛𝑔𝑒 𝑖𝑛 𝑥 − 𝑐𝑜𝑜𝑟𝑑𝑖𝑛𝑎𝑡𝑒 Δ𝑥 𝑥2 − 𝑥1
Example
Calculate the slope of a line given the following two points on the line,
(2,1) and (−4, −2)
Solution
The slope, m is calculated using the equation above.
𝛥𝑦 𝑦2 − 𝑦1 −2 − (−1) −3 1
𝑚= = = = = = 0.5
𝛥𝑥 𝑥2 − 𝑥1 −4 − (−2) −6 2
This is exactly the same result that you would obtain if you plotted these
points on a graph and measured the slope directly from the graph as
height, 3, divided by distance, 6

4.1.2.5 The equation of a line given slope and any point on


𝒚 — 𝒚𝟏 = 𝒎(𝒙 — 𝒙𝟏 ) the line
The equation of a Since the slope of a line is the same when measured between any two
points, such as the points (𝑥1 , 𝑦1 ) and (𝑥, 𝑦), we may write
straight line with
slope, m, which passes Δ𝑦 𝑦 − 𝑦1
=𝑚=
through the specific Δ𝑥 𝑥 − 𝑥1
point (𝑥1 , 𝑦1 ) therefore, 𝑚(𝑥 — 𝑥1 ) = 𝑦 — 𝑦1 . This equation is usually written as:
𝑦 — 𝑦1 = 𝑚(𝑥 — 𝑥1 ).
Example
Find the equation of a line having a slope of 1.7 and which passes
through the point (2, 5).
Solution
Using equation 𝑦 — 𝑦1 = 𝑚(𝑥 — 𝑥1 ), the equation of the line is
deduced as
𝑦 − 5 = 1.7(𝑥 − 2)
𝑦 − 5 = 1.7𝑥 − 3.4

73
74 Unit 4: Types of Functions

𝑦 = 1.7𝑥 + 1.6

4.1.2.6 The equation of a line given two points


There are two steps involved when deducing the equation of a line when
given two points, say points (𝑥1 , 𝑦1 ) and (𝑥2 , 𝑦2 ).
𝑦 −𝑦
Step 1: Calculate the slope of the line using equation 𝑚 = 𝑥2 −𝑥1 .
2 1

Step 2: Determine the equation of the line by substituting the slope (as
calculated in step 1), and either point into equation 𝑦 — 𝑦1 =
𝑚(𝑥 — 𝑥1 ).
Example
a) A line passes through the points (2, 4) and (6, 1). Deduce the
equation of the line.
b) Plot the graph of the line.
Solution
(a) Step 1: The slope of the line is calculated by substituting the points
into the equation
𝑦2 − 𝑦1 1 − 4 3
𝑚= = = − = −0.75
𝑥2 − 𝑥1 6 − 2 4

Step 2: The equation of the line is deduced by substituting this slope, and
either point into equation:
𝑦 — 𝑦1 = 𝑚(𝑥 — 𝑥1 ).
𝑦 − 4 = −0.75(𝑥 − 2) using point (2, 4)
𝑦 − 4 = −0.75. 𝑥 + 1.5
𝑦 = 5.5 − 0.75𝑥
The equation of the line has a negative slope of —0.75 and y-intercept of
5.5.
Hence, if the slope and intercept of a line are given, the equation of that
line may be obtained by substituting the given values of m and c into the
general formula 𝑦 = 𝑚𝑥 + 𝑐.
Examples
1. Determine the slope and intercept of the straight line 9x + 3y = 4.
Solution. We need to write this equation in the form 𝑦 = 𝑚𝑥 + 𝑐.
9𝑥 + 3𝑦 = 4
3𝑦 = −9𝑥 + 4
4
𝑦 = −3𝑥 +
3
BBA 120 Business Mathematics

One can say immediately that the slope of this straight line is −3 and the
4
intercept is 3 .
2. Plot the lines
a) y=x
b) y=x+2
3. Find the slope of the straight line that passes through the points (2, −1)
and (−2, −11).
Solution. The slope of a straight line passing through the points (x1, y1),
(x2, y2) is
𝑦2 − 𝑦1
𝑚=
𝑥2 − 𝑥1
Therefore the required slope is
−11 − (−1) 10 5
𝑚 = =− =
−2 − 2 −4 2

Exercises
1. Sketch the graph of the straight line 𝑦 = −𝑥 + 2 for −1 ≤
𝑥 ≤ 5.
2. Sketch the graph of the straight line 𝑦 = 2𝑥 − 3 for 0 ≤ 𝑥 ≤
4.
3. Find the slope of the straight line passing through the points
(−1,−3) and (4, 2).
4. Given the line, 𝑦 = 65 − 2𝑥
a) Set up a table of points, hence plot the line over the
interval 0 < 𝑥 < 34.
b) Write down values of and describe verbally the slope
and intercepts.
5. Given the equations of the following lines:
(a) 2𝑦 − 5𝑥 + 10 = 0 (b) 𝑥 = 10 − 2𝑦 (c) 𝑦 + 5𝑥 = 15

i. Write each of the equations in the form y = f (x).


ii. Calculate the intercepts. Plot each line by joining the
intercepts.
From the graph, how would you determine whether the slope is
positive or negative?

75
76 Unit 3: Functions and Graphs

Unit 3: Functions and Graphs

4.2 Applications: Demand,


Supply, Costs, Revenue
4.2.1 Demand and Supply
The demand function
For each price level of a product,
there is a corresponding quantity
of that product that consumers
will demand (that is, purchase)
during some time period.
Usually, the higher the price, the
smaller is the quantity demanded;
as the price falls, the quantity
demanded increases. If the price
per unit of the product is given by
P and the corresponding quantity
(in units) is given by Q, then an
equation relating P and Q is called
a demand equation.
There are several variables that influence the demand for a good X. These
may be expressed by the general demand function
Q = f(P, Y, Ps, Pc, Ta, A,...)
where
Q is the quantity demanded of good X
P is the price of good X
Y is the income of the consumer
PS is the price of substitute goods
Pc is the price of complementary goods
Ta is the taste or fashion of the consumer
A is the level of advertising

The simplest model for the demand function is written as


Q = f(P)
Therefore, quantity demanded depends on price only, so long as the other
variables upon which demand depends remain constant; that is, Y, Ps, Pc,
Ta, A,... are constant.
BBA 120 Business Mathematics

For example, the demand for good X may be given by the linear equation,
𝑸 = 𝟐𝟎𝟎 − 𝟐𝑷.

This equation describes the law of demand, a basic economic hypothesis


which states that there is a negative relationship between quantity
demanded and price, that is, when the price of a good increases, the
quantity demanded will decrease, all other variables remaining constant.

Demand functions (a) Q = f ( P ) and (b) P = g(Q)

Example: Finding a Demand Equation


Suppose the demand per week for a product is 100 units when the price is
$58 per unit and 200 units at $51 each. Determine the demand equation,
assuming that it is linear.
Solution:
Strategy Since the demand equation is linear, the demand curve must be a
straight line. We are given that quantity q and price p are linearly related
such that p = 58 when q = 100 and p = 51 when q = 200. Thus, the given
data can be represented in a (q, p)-coordinate plane by points (100, 58)
and (200, 51). With these points, we can find an equation of the line-that
is, the demand equation.
The slope of the line passing through (l00, 58) and (200, 51) is
51 − 58 7
𝑚= = −
200 − 100 100
An equation of the line (point-slope form) is
𝑝 − 𝑝1 = 𝑚(𝑞 − 𝑞1 )

77
78 Unit 3: Functions and Graphs

7
𝑝 − 58 = − (𝑞 − 100)
100
Simplifying gives the demand equation
7
𝑝=− 𝑞 + 65
100
Equilibrium Price
Microeconomics is concerned with the analysis of the economic theory
and policy of individual firms and markets. The mathematics we have
introduced so far can be used to calculate the market equilibrium in
which the demand and supply of a particular good balance.

We are interested in the interplay between supply and demand. Of


particular significance is the point of intersection of the demand and
supply curves (see the figure above).
At this point, the market is said to be in equilibrium because the quantity
demanded is equal to the quantity supplied. The corresponding price, P,
and quantity, Q, are called the equilibrium price and quantity.
It is also of interest to observe the effect of a shift of the market price
away from its equilibrium price.

Example
The demand and supply functions of a good are given by
𝑃 = −𝑄𝑑 + 125,
2𝑃 = 3𝑄𝑠 + 30.
a) Determine the equilibrium price and quantity.
b) Determine also the effect on the market equilibrium if the government
decides to impose a fixed tax of £5 on each good.
c) Who pays the tax?
Solution.
a) At market equilibrium, we have
𝑄𝑑 = 𝑄𝑠 = 𝑄,
BBA 120 Business Mathematics

here Q is the equilibrium quantity. In this case, the demand and supply
equations become
𝑃 = −𝑄 + 125, (𝑖)
2𝑃 = 3𝑄 + 30. (𝑖𝑖)
This is a system of two simultaneous equations in the unknowns P and Q.
We can eliminate Q from the system by multiplying the demand equation
(Eq.(i)) by 3:
3𝑃 = −3𝑄 + 375, (𝑖𝑖𝑖)
and adding the resulting equation (i) to the supply equation (ii). This
gives
5𝑃 = 405,
which, after dividing both sides by 5 gives the equilibrium price
𝑃 = 81.
Finally, the equilibrium quantity Q is determined by substituting this
value into either of the demand or supply equations. The demand
equation gives
81 = −𝑄 + 125,
which, after rearrangement yields the equilibrium quantity
𝑄 = 125 − 81 = 44.

b) If the government imposes a fixed tax of £5 on each good, then the


original supply equation needs to be modified.
This is because the amount the supplier receives as a result of each sale is
the amount that the consumer pays (P) less the tax (£5), i.e., P −5. Thus,
the new supply equation is obtained by replacing P by P − 5 in the
original supply equation:
2(𝑃 − 5) = 3𝑄𝑠 + 30. (𝑖𝑣)
This equation can be simplified by multiplying out the bracket on the left
hand side and taking the constant term to the right-hand side. The new
supply equation becomes
2𝑃 − 10 = 3𝑄𝑠 + 30,
or
2𝑃 = 3𝑄𝑠 + 40. (𝑣)
We then proceed as before to determine the equilibrium price and
quantity for the new situation. At market equilibrium, we have
𝑄𝑑 = 𝑄𝑠 = 𝑄,
where Q is the equilibrium quantity. In this case, the demand and supply
equations become
𝑃 = −𝑄 + 125, (𝑣𝑖)

79
80 Unit 3: Functions and Graphs

2𝑃 = 3𝑄 + 40. (𝑣𝑖𝑖)
We can eliminate Q from the system by multiplying the demand equation
(Eq. (vi)) by 3:
3𝑃 = −3𝑄 + 375, (𝑣𝑖𝑖𝑖)
and adding the resulting equation (viii) to the supply equation (vii). This
gives
5𝑃 = 415,
which, after dividing both sides by 5 gives the equilibrium price
𝑃 = 83.
Finally, the equilibrium quantity Q is determined by substituting this
value into either of the demand or supply equations. The demand
equation gives
83 = −𝑄 + 125,
which, after rearrangement yields the equilibrium quantity
𝑄 = 125 − 83 = 42.
The influence of government taxation on the equilibrium price is to
increase it from £81 to £83.
c) Therefore, not of all of the tax is passed on to the consumer. The
consumer pays an extra £2 per good after tax has been imposed. The
remaining part of the tax is borne by the supplier.

Exercises
1. Suppose consumers will demand 60 units of a product when the price is
$15.30 per unit and 35 units when the price is $19.30 each. Find the demand
equation, assuming that it is linear. Find the price per unit when 40 units are
demanded.
2. The demand per week for a CD is 26,000 copies when the price is $12 each,
and 10,000 copies when the price is $18 each. Find the demand equation for
the CD assuming that it is linear.
3. A refrigerator manufacturer will produce 3000 units when the price is $940,
and 2200 units when the price is $740. Assume that price, p, and quantity,
q, produced are linearly related and find the supply equation.
4. Supply Equation Suppose a manufacturer of shoes will place on the market
50 (thousand pairs) when the price is 35 (dollars per pair) and 35 when the
price is 30. Find the supply equation, assuming that price p and quantity q
are linearly related.

4.2.2 Cost
Firms incur costs when they employ inputs, such as capital and labour, in
order to produce goods for sale in the market. The total cost of producing
a good will normally consist of:
BBA 120 Business Mathematics

(i) Fixed costs, FC: costs that are fixed irrespective of the level of output,
e.g. rent on premises.
(ii) Variable costs, VC: costs which vary with the level of output, e.g. each
extra unit of a good produced will require additional units of raw
materials, labour, etc.
Total cost, TC, is therefore the sum of fixed costs and variable costs:
𝑇𝐶 = 𝐹𝐶 + 𝑉𝐶
These costs can be modelled by a linear cost function, such as
𝑇𝐶 = 20 + 4𝑄
where FC = 20 (the vertical intercept) and VC = 4Q, where 4 = slope of
the line.
Example
A firm has fixed production costs of £10 and variable production costs of
£2 per unit produced.
(a) Write down the equation of the total cost function.
(b) Graph the total cost function.
Solution
a) FC = £10; while VC = £2Q, since to produce
1 unit PC = 2(1)
2 units VC = 2(2)
3 units VC = 2(3)
Q units VC = 2(0)
that is, the total variable costs incurred in producing Q units of this good
is £2Q.
Since TC = FC + VC, the total cost incurred in producing Q units is
TC = 10 + 2Q
b) Graphically, the TC function is a straight line, with costs measured on the
vertical axis and units of the good produced on the horizontal axis.
Since the vertical intercept = 10 (level of FC) and the slope = 2 are
known, the total cost function may be plotted. Alternatively, total costs
may be plotted by calculating at least two points on the line and joining
these points.

4.2.3 Revenue
A firm receives revenue when it sells output. The total revenue, TR,
received is the price of the good, P, multiplied by the number of units
sold, Q, that is,
𝑇𝑅 = 𝑃0 𝑄

81
82 Unit 3: Functions and Graphs

where P0 is the constant price per unit of the good and is represented by
the vertical intercept of a horizontal demand function.
Example
Suppose that each chicken snack box is sold for £3.50 irrespective of the
number of units sold.
a) Write down the equation of the total revenue function.
b) Graph the total revenue function.
Solution
a) Total revenue is price multiplied by the number of units sold, that is,
𝑇𝑅 = 3.5𝑄
Note that price is constant at £3.50 irrespective of the value of Q.
b) Total revenue is represented graphically by a straight line, with slope =
3.5 and intercept = 0. It is graphed by calculating values of TR for any
values of Q, for example Q = 0,2,4,6.

Exercises
1. A firm faces the following cost function, TC = 5Q.
(a) What is the value of fixed costs?
(b) Graph the total cost function.
(c) What is the total cost when Q = 10?
(d) What is the cost of producing each additional unit of this good?
2. A firm sells its product in a perfectly competitive output market (i.e.
the price is the same for each unit sold). The total revenue function is
given as TR = 10Q.
(a) What is the price per unit charged by the perfectly competitive
firm?
(b) Plot the total revenue function.
3. The canoeing club provides swimming lessons to boost club funds. The
club has fixed costs of £250 daily (insurance) when offering these
lessons. The variable cost is £25 for each lesson given.
(a) Write down the equation for total cost and plot its graph for Q = 0
to 60.
(b) Calculate the cost of providing 28 lessons.
(c) Calculate the number of lessons provided when total costs are £1400.
(d) Confirm the answers in (b) and (c) graphically.
BBA 120 Business Mathematics

4.2.3 Elasticity of Demand, Supply and Income

The concept of elasticity, as used in economics,


is the ratio of the percentage change in the
quantity demanded (or supplied) to a percentage
change in an economic variable, such as price,
income, etc.
It is a measure of the relationship between a
change in the quantity demanded of a particular
good and a change in its price.
Price elasticity of demand is a term in economics
often used when discussing price sensitivity. The
formula for calculating price elasticity of demand
is:

% 𝑪𝒉𝒂𝒏𝒈𝒆 𝒊𝒏 𝑸𝒖𝒂𝒏𝒕𝒊𝒕𝒚 𝑫𝒆𝒎𝒂𝒏𝒅𝒆𝒅


Price Elasticity of Demand =
% 𝑪𝒉𝒂𝒏𝒈𝒆 𝒊𝒏 𝑷𝒓𝒊𝒄𝒆

If a small change in price is accompanied by a large change in quantity


demanded, the product is said to be elastic (or responsive to price
changes).
Conversely, a product is inelastic if a large change in price is
accompanied by a small amount of change in quantity demanded.
Elasticity may be used to predict the responsiveness of demand (and
supply) to changes in such economic variables.
There are various symbols used to denote elasticity. In this text it is
denoted by the symbol ε, pronounced 'epsilon'.
Three types of elasticity are introduced: price elasticity of demand, εd,
price elasticity of supply, εs, and income elasticity of demand, εy.

Price elasticity of demand


Price elasticity of demand measures the responsiveness (sensitivity) of
quantity demanded to changes in the good's own price. It is also referred
to as own-price elasticity. It is represented by the general elasticity
formula
𝛥𝑄
% 𝑐ℎ𝑎𝑛𝑔𝑒 𝑖𝑛 𝑞𝑢𝑎𝑛𝑡𝑖𝑡𝑦 𝑑𝑒𝑚𝑎𝑛𝑑𝑒𝑑 %𝛥𝑄𝑑 ∙100
𝑄
𝜀𝑑 = % 𝑐ℎ𝑎𝑛𝑔𝑒 𝑖𝑛 𝑝𝑟𝑖𝑐𝑒
= %𝛥𝑃
= 𝛥𝑃
∙100
𝑃

which simplifies to
Price Elasticity 𝛥𝑄 𝑃
of Demand 𝜀𝑑 = =
(PED)
Δ𝑃 𝑄
measures the The numerical value or coefficient for price elasticity of demand is
responsiveness normally negative since Δ Q / Δ P is negative. That is, the linear demand
of demand function, 𝑃 = 𝑎 − 𝑏𝑄 has slope
after a change
in price

83
84 Unit 3: Functions and Graphs

Δ𝑃 𝑏
= =−
Δ𝑄 1
hence, inverting both sides gives
Δ𝑄 1
= =−
Δ𝑃 𝑏
which is negative.
Point elasticity of demand
Given the linear demand function, 𝑃 = 𝑎 − 𝑏𝑄, the formula for point
elasticity of demand at any point (P0,Q0) is
𝛥𝑄 𝑃0
𝜀𝑑 =
𝛥𝑃 𝑄0
Or
1𝑃 𝛥𝑄 1
𝜀𝑑 = − 𝑏 𝑄0 since slope = 𝛥𝑃
= −𝑏
0

An economist Example
is an expert Given the demand function for computers as P = 2400 - 0.5Q
who will know a) Determine the coefficient of point elasticity of demand when
tomorrow why (i) P = 1800,
the things he
(ii) P = 1200 and
predicted
yesterday (iii) P = 600.
didn't happen Give a verbal description of each result.
today. b) If the price of computers increases by 12%, calculate the percentage
change in the quantity demanded at P = 1800, P = 1200 and P = 600.
Laurence J. Peter
(1919 - 1988) (i) First use the definition of elasticity:
% 𝑐ℎ𝑎𝑛𝑔𝑒 𝑖𝑛 𝑞𝑢𝑎𝑛𝑡𝑖𝑡𝑦 𝑑𝑒𝑚𝑎𝑛𝑑𝑒𝑑 𝛥𝑄𝑑 (%)
𝜀𝑑 = =
% 𝑐ℎ𝑎𝑛𝑔𝑒 𝑖𝑛 𝑝𝑟𝑖𝑐𝑒 𝛥𝑃 (%)
(ii) Then calculate the exact percentage changes and compare them
with the answers in (i).
c) Graph the demand function, indicating where demand is elastic, unit
elastic and inelastic.
Solution
(a)
(i) At P = 1800 the quantity of computers demanded, Q, is calculated by
substituting P = 1800 into the demand function:
𝑃 = 2400 − 0.5𝑄
1800 = 2400 − 0.5𝑄
0.5𝑄 = 600
𝑄 = 1200
BBA 120 Business Mathematics

The value of point elasticity of demand at (P = 1800, Q = 1200) is


calculated by substituting these values along with b = 0.5 into formula
1𝑃
𝜀𝑑 = − 𝑏 𝑄0 :
0

1 1800 1800 3
=− × =− = − = −3
0.5 1200 600 1
The coefficient of point elasticity of demand is 𝜀𝑑 = −3, which indicates
that at the price P = 1800 a 1% increase (decrease) in price will cause a
3% decrease (increase) in the quantity of computers demanded.
Demand is elastic, |𝜀𝑑 | > 1.
(ii) and (iii) The calculations for price elasticity of demand for P = 1200
and p = 600 are carried out in the same way as (i) and are summarised
below.
εd for P=1200 is -1 thus Demand is unit elastic |𝜀𝑑 | = 1.
εd for P=600 is -1/3 thus Demand is inelastic |𝜀𝑑 | < 1.
Notes
εd = — 1 indicates that at P = 1200, a 1% increase (decrease) in price
will cause a 1% decrease (increase) in the quantity of computers
demanded.
εd = -0.33 (correct to 2 decimal places) indicates that at P = 600, a 1 %
increase (decrease) in price will cause a 0.33% decrease (increase) in
the quantity of computers demanded.
b) The definition
% 𝑐ℎ𝑎𝑛𝑔𝑒 𝑖𝑛 𝑞𝑢𝑎𝑛𝑡𝑖𝑡𝑦 𝑑𝑒𝑚𝑎𝑛𝑑𝑒𝑑 %𝛥𝑄𝑑
𝜀𝑑 = =
% 𝑐ℎ𝑎𝑛𝑔𝑒 𝑖𝑛 𝑝𝑟𝑖𝑐𝑒 %𝛥𝑃
may be rearranged as
Δ𝑄 (%) = Δ𝑃 (%) × 𝜀𝑑
 At P = 1800 we have calculated that εd = -3. To calculate the
percentage change in Q, substitute εd = -3 and ΔP = 12% into
equation 𝛥𝑄 (%) = 𝛥𝑃 (%) × 𝜀𝑑 :
Δ𝑄 = 12% × −3 = −36%
The quantity demanded decreases by 36%, a larger percentage
decrease than the 12% price increase.
Demand is strongly responsive to price change and is described as
elastic demand.
 P = 1200: εd = -1, AP = 12%. Substitute into equation 𝛥𝑄 (%) =
𝛥𝑃 (%) × 𝜀𝑑 :
𝛥𝑄 = 12% × −1 = −12%
The quantity demanded decreases by 12%; the percentage increase
in price is 12%. This is described as unit elastic demand.

85
86 Unit 3: Functions and Graphs

 P = 600: εd = -0.33, AP = 12%. Substitute into equation 𝛥𝑄 (%) =


𝛥𝑃 (%) × 𝜀𝑑 :
𝛥𝑄 = 12% × −0.33 = −3.96%
The quantity demanded decreases by 3.96%, a smaller percentage
decrease than the 12% price increase. So demand is weakly
responsive to price, hence it is described as inelastic demand.
(ii) The calculation of the exact percentage change in Q requires basic
arithmetic. Start by calculating the price which results from an increase of
12% on the initial price.
112
Increase P = 1800 by 12% → Pnew = 100 𝑃 = 1.12(1800) = 2016
Then calculate the corresponding values of Q from the equation of the
demand function.
When P = 1800, Q = 1200 and when P = 2016, Q = 768.
We find calculate the direct percentage change as
768 − 1200
× 100 = −36%
1200
We apply a similar procedure for (P,Q)=(1200,2400) and (600,3600)
c) The graph of the demand function is given below.

Price elasticity of supply


The same ideas as developed with respect to the price elasticity of
demand can be applied to the analysis of the price elasticity of supply at a
point on the supply function or averaged along an arc on the supply
BBA 120 Business Mathematics

function. As the supply function is positively sloped, the coefficient of


price elasticity of supply will be positive.
The coefficient of price elasticity of supply is given by the formula
% 𝑐ℎ𝑎𝑛𝑔𝑒 𝑖𝑛 𝑞𝑢𝑎𝑛𝑡𝑖𝑡𝑦 𝑠𝑢𝑝𝑝𝑙𝑖𝑒𝑑 %𝛥𝑄𝑠 Δ𝑄𝑠 𝛥𝑄 𝑃
𝜀𝑠 = = = = ∙
% 𝑐ℎ𝑎𝑛𝑔𝑒 𝑖𝑛 𝑝𝑟𝑖𝑐𝑒 %𝛥𝑃 Δ𝑃 𝛥𝑃 𝑄

Given the supply function, P = c + dQ, slope = d = ΔP/ΔQ.


Then, 1/d = ΔQ/ΔP; hence, the point elasticity of supply formula is
1 P
εs = ∙
d Q
Note: εs > 1: supply is elastic; εs < 1: supply is inelastic; εs = 1: supply is
unit elastic.

Income elasticity of Demand


Income elasticity of demand is a measure of how much demand for a
good/service changes relative to a change in income, with all other
factors remaining the same.
The formula for income elasticity is:
% 𝑐ℎ𝑎𝑛𝑔𝑒 𝑖𝑛 𝑞𝑢𝑎𝑛𝑡𝑖𝑡𝑦 𝑑𝑒𝑚𝑎𝑛𝑑𝑒𝑑
𝐼𝑛𝑐𝑜𝑚𝑒 𝐸𝑙𝑎𝑠𝑡𝑖𝑐𝑖𝑡𝑦 =
% 𝑐ℎ𝑎𝑛𝑔𝑒 𝑖𝑛 𝑖𝑛𝑐𝑜𝑚𝑒
For example, if, in response to a 10% increase in income, the demand for
a good increased by 20%, the income elasticity of demand would be
20%/10% = 2.

4.2.4 Budget Constraints

A budget constraint represents all the combinations of goods and


services that a consumer may purchase given current prices within his or
her given income.
Suppose that a company or an individual has a given budget, B, that can
be used to purchase two goods. If the cost or price of each of these goods
is known, then it is possible to determine the different combinations of
the two goods that can be bought with the given budget.
Suppose that the two goods are denoted by X and Y , and their respective
prices are PX and PY. The quantities purchased of these goods is also
denoted by X and Y . Then the equation of the budget line is
𝑃𝑋 𝑋 + 𝑃𝑌 𝑌 = 𝐵

87
88 Unit 3: Functions and Graphs

Example
An electrical company has a budget of £6,000 a week to spend on the
manufacture of toasters and kettles. It costs £5 to manufacture a toaster
and £12 to manufacture a kettle. Write down the equation of the budget
line and sketch its graph.
Solution.
Let T and K denote the number of toasters and kettles that are
manufactured each week. Then the cost of manufacture and the available
budget means that the budget line has the equation
5𝑇 + 12𝐾 = 6000

To sketch the graph of this budget line, it is sufficient to determine the


coordinates of two points on the line. When T = 0, 12K = 6,000 and
therefore K = 500. Similarly, when K = 0, 5T = 6,000 and therefore T =
1,200.
The graph of the budget line is given by the straight line joining the
points T = 0, K = 500 and T = 1,200, K = 0. The graph of the budget line
is sketched in the figure above.

Exercise
A person has £120 to spend on two goods (X, Y ) whose respective prices
are £3 and £5.
a) Draw a budget line showing all the different combinations of the
two goods that can be bought with the given budget (B).
b) What happens to the original budget line if the budget falls by
25%?
c) What happens to the original budget line if the price of X
doubles?
BBA 120 Business Mathematics

d) What happens to the original budget line if the price of Y falls to


£4?
Draw the new budget lines in each case.

Unit 4: Types of Functions

4.2 Simultaneous Equations


When a situation must be described mathematically, it is not unusual for a
set of equations to arise. We call this set of equations a system of two
linear equations in the variables x and y. The problem is to find values of
x and y for which both equations are true simultaneously. A pair (x, y) of
such values is called a solution of the system.
In this section we shall learn how to solve a systems of linear equations in
both two and three variables by using the technique of elimination by
addition or by substitution.

4.2.1 Elimination-by-Addition Method


Example
1. Use elimination by addition to solve the system.
3𝑥 − 4𝑦 = 13 … … … … … … … … … … … … … … … . . (1)
{
3𝑦 + 2𝑥 = 3 … … … … … … … … … … … … … … … . . … (2)
Solution:
Aligning the x- and y-terms for convenience gives
3𝑥 − 4𝑦 = 13
{
2𝑥 + 3𝑦 = 3
To eliminate y, we multiply Equation (1) by 3 and Equation (2) by 4:
9𝑥 − 12𝑦 = 39 … … … … … … … … … … . . … … … … … . . (3)
{
8𝑥 + 12𝑦 = 12 … … … … … … … … … … … … … … … . . … (4)
Adding Equation (3) to Equation (4) gives 17𝑥 = 51, from which 𝑥 =
3.
Replacing x by 3 in Equation (3) results in 𝑦 = −1.
The solution is x = 3 and y = -1. The Figure on the left shows a graph of
the system.

2. Given the simultaneous equations

89
90 Unit 4: Types of Functions

𝑥 + 3𝑦 = 4 … … … … … … … … … … . . … … … . . … … . . (1)
{
−𝑥 + 2𝑦 = 6 … … … … … … … … … … … … … … … . . … (2)
(a) Solve for x and y algebraically.
(b) Solve for x and y graphically.
Solution
(Clue)
Eliminate x from the system of equations by adding equations (1)and (2). The
two equations reduce to a single equation in which the only unknown is v. Solve
for v, then substitute the value of v into either of the original equations and solve
for x.

4.2.2 Method of Elimination by Substitution


Example
1. Use elimination by substitution to solve the system
𝑥 + 2𝑦 − 8 = 0 … … … … … … … … … … . . … … … . . … … . . (1)
{
2𝑥 + 4𝑦 + 4 = 0 … . … … … … … … … … … … … … … … . . … (2)

Solution
It is easy to solve the first equation for x. Doing so gives the equivalent
system
𝑥 = −2𝑦 + 8 … … … … … … … … … … . . … … … . . … … . . (3)
{
2𝑥 + 4𝑦 + 4 = 0 … . … … … … … … … … … … … … … … . . … (4)
Substituting −2𝑦 + 8 for x in Equation (4) yields
2(−2𝑦 + 8) + 4𝑦 + 4 = 0
−4𝑦 + 16 + 4𝑦 + 4 = 0
The latter equation simplifies to 20 = 0. Thus, we have the
system
𝑥 = −2𝑦 + 8 … … … … … … … … … … . . … … … . . … . . … . . (5)
{
20 = 0 … . … … … … … … … … … . … … … … … … … … … . . … (6)
Since Equation (6) is never true, there is no solution of the original
system.
2. A chemical manufacturer wishes to fill an order for 500 liters of a 25%
acid solution. (Twenty-five percent by volume is acid.) If solutions of
30% and 18% are available in stock, how many liters of each must be
mixed to fill the order?
Solution
Solution: Let x and y be the number of liters of the 30% and 18%,
solutions respectively, that should be mixed. Then
𝑥 + 𝑦 = 500
In 500 liters of a 25% solution, there will be 0.25(500) = 125 liters of
acid. This acid comes from two sources: 0.30𝑥 liters of it come from the
30% solution, and 0.18𝑦 liters of it come from the 18% solution. Hence,
BBA 120 Business Mathematics

0.30𝑥 + 0.18𝑦 = 125


These two equations form a system of two linear equations in two
unknowns.

Solving the first for x gives, x = 500 - y.


Substituting in the second gives
0.30(500 − 𝑦) + 0.18𝑦 = 125
1
Solving this equation for y, we find that 𝑦 = 208 3 liters.
1 2
Thus, 𝑥 = 500 − 208 3 = 291 3 liters.

4.3 Quadratic Functions Revisited.


Recall that a quadratic function is a polynomial function of the form
𝑓(𝑥) = 𝑎𝑥 2 + 𝑏𝑥 + 𝑐, 𝑎 ≠ 0.
The graph of a quadratic function is a parabola whose axis of symmetry is
parallel to the y-axis.
The expression 𝑎𝑥 2 + 𝑏𝑥 + 𝑐 in the definition of a quadratic function is a
polynomial of degree 2 or second order, or a 2nd degree polynomial,
because the highest exponent of x is 2.
If the quadratic function is set equal to zero, then the result is a quadratic
equation.
The solutions to the equation are called the roots of the equation.

4.3.1 Sketching a quadratic function


The general technique for graphing quadratics is the same as for graphing
linear equations. However, since quadratics graph as curvy lines (called
"parabolas"), rather than the straight lines generated by linear equations,
there are some additional considerations.
The most basic quadratic is y = x2. When you graphed straight lines, you
only needed two points to graph your line, though you generally plotted
three or more points just to be on the safe side. However, three points will
almost certainly not be enough points for graphing a quadratic function.

91
92 Unit 4: Types of Functions

You find many points:

That last point has a rather large y-value, so you decide that you won't
bother drawing your graph large enough to plot it.

But you plot all the other points:

You draw a nicely smooth curving line passing neatly through the plotted
points:

correct graph of y = x2
BBA 120 Business Mathematics

The general form of a quadratic is "𝑦 = 𝑎𝑥 2 + 𝑏𝑥 + 𝑐". For graphing,


the leading coefficient "a" indicates how "fat" or how "skinny" the
parabola will be.
For | a | > 1 (such as a = 3 or a = –4), the parabola will be "skinny",
because it grows more quickly (three times as fast or four times as fast,
respectively, in the case of our sample values of a).
For | a | < 1 (such as a = 1/3 or a = –1/4 ), the parabola will be "fat",
because it grows more slowly (one-third as fast or one-fourth as fast,
respectively, in the examples).
Also, if a is negative, then the parabola is upside-down.
There is a simple, if slightly "dumb", way to remember the difference
between right-side-up parabolas and upside-down parabolas:

positive quadratic y = x2 negative quadratic y = –x2

93
94 Unit 4: Types of Functions

Parabolas always have a lowest point (or a highest point, if the parabola
is upside-down). This point, where the parabola changes direction, is
called the "vertex".
If the quadratic is written in the form y = a(x – h)2 + k, then the vertex is
the point (h, k).
(Note: The "a" in the vertex form "y = a(x – h)2 + k" of the quadratic is
the same as the "a" in the common form of the quadratic equation, "y =
ax2 + bx + c".)

Since the vertex is a useful point, and since you can "read off" the
coordinates for the vertex from the vertex form of the quadratic, you can
see where the vertex form of the quadratic can be helpful, especially if
the vertex isn't one of your T-chart values. However, quadratics are not
usually written in vertex form.
You can complete the square to convert ax2 + bx + c to vertex form, but,
for finding the vertex, it's simpler to just use a formula. (The vertex
formula is derived from the completing-the-square process, just as is the
Quadratic Formula. In each case, memorization is probably simpler than
completing the square.)
For a given quadratic y = ax2 + bx + c, the vertex (h, k) is found by
computing h = –b/2a, and then evaluating y at h to find k.
If you've already learned the Quadratic Formula, you may find it easy to
memorize the formula for k, since it is related to both the formula for h
and the discriminant in the Quadratic Formula:
(4𝑎𝑐 – 𝑏2)
𝑘 = .
4𝑎
Example
Find the vertex of y = 3x2 + x – 2 and graph the parabola.
Solution
To find the vertex, I look at the coefficients a, b, and c. The formula for
the vertex gives me:
𝑏 1 1
ℎ =– =– =–
2𝑎 2(3) 6
BBA 120 Business Mathematics

Then I can find k by evaluating y at h = –1/6:


k = 3( –1/6 )2 + ( –1/6 ) – 2
= 3/36 – 1/6 – 2
= 1/12 – 2/12 – 24
/12
–25
= /12

So now I know that the vertex is at ( –1/6 , –25/12 ). Using the formula was
helpful, because this point is not one that I was likely to get on my T-
chart.

I need additional points for my


graph:

95
96 Unit 4: Types of Functions

Now I can do my graph, and I will


label the vertex:

Example
Find the x-intercepts and vertex of y = –x2 – 4x + 2.
Solution
Since it is so simple to find the y-intercept (and it will probably be a point
in my T-chart anyway), they are only asking for the x-intercepts this time.
To find the x-intercept, I set y equal 0 and solve:
0 = –x2 – 4x + 2
x2 + 4x – 2 = 0

For graphing purposes, the intercepts are at about (–4.4, 0) and (0.4, 0).
(When I write down the answer, I will of course use the "exact" form,
with the square roots; my calculator's decimal approximations are just for
BBA 120 Business Mathematics

helping me graph.)

To find the vertex, I look at the coefficients: a = –1 and b = –4. Then:


h = –(–4)/2(–1) = –2
To find k, I plug h = –2 in for x in y = –x2 – 4x + 2, and simplify:
k = –(–2)2 – 4(–2) + 2 = –4 + 8 + 2 = 10 – 4 = 6
Now I'll find some additional plot points, to help me fill in my graph:

Note that I picked x-values that were centered around the x-coordinate of
the vertex. Now I'll plot the parabola:

The vertex is at (–2, 6), and the intercepts are at the following points:

(0, 2), , and

97
98 Unit 4: Types of Functions

Example
1. The number of bacteria in a refrigerated food is given by𝑁(𝑇) = 20𝑇 2 −
20𝑇 + 120, for −2 ≤ 𝑇 ≤ 14 and where T is the temperature of the food
in Celsius. At what temperature will the number of bacteria be minimal?
Solution
First we can see that we have a quadratic function given to us. Moreover,
the parabola would open up. So that means that the vertex of the
parabola represents a minimum value. This means the first thing we need
to do is determine the vertex, since we want the minimal number of
bacteria.
𝑏
As we stated, we will use the formula 𝑥 = – to find the vertex since it
2𝑎
is much more efficient than the completing the square method for finding
the vertex in word problems. Since our function is in terms of T, the
𝑏
formula we really use is 𝑇 = – . So we get
2𝑎
−20 20 1
𝑇 = – 2(20) = 40
=2

So the T value of the vertex is ½. Now we must look back at the question to see
what we really wanted. Since we want the temperature at which the number is
minimum and T is the temperature in Celsius, this is the value we want.
Therefore, the minimum number of bacteria are present when the temperature is
0
½ Celsius.
2. Given the supply and demand functions
𝑃 = 𝑄𝑠2 + 12𝑄𝑠 + 32,
𝑃 = −𝑄𝑑2 − 4𝑄𝑑 + 200,
calculate the equilibrium price and quantity.
Solution
Solution. At equilibrium, the quantity supplied is equal to the quantity
demanded,
so that
𝑄𝑑 = 𝑄𝑠 = 𝑄, say.
Then the supply and demand equations become
𝑃 = 𝑄 2 + 12𝑄 + 32,
𝑃 = −𝑄 2 − 4𝑄 + 200.
Equating the expressions on the right-hand sides of these equations, we
have
𝑄 2 + 12𝑄 + 32 = −𝑄 2 − 4𝑄 + 200.
We can do this since both expressions are equal to P. Rearranging this
equation and collecting like terms yields the quadratic equation
2𝑄 2 + 16𝑄 − 168 = 0.
This equation can be simplified by dividing throughout by 2. We then
have the
BBA 120 Business Mathematics

quadratic equation 𝑄 2 + 8𝑄 − 84 = 0.
Solving this equation using the formula with a = 1, b = 8, and c = −84
yields
−8 ± 20
𝑄=
2
Therefore, either Q=6 or Q=-14
The solution 𝑄 = −14 can be discarded because a negative quantity
does not make sense.
Therefore, the equilibrium quantity is 6. The corresponding equilibrium
price can be determined by substituting Q = 6 into either the supply or
demand equation. If we substitute this value into the supply equation, we
have
𝑃 = 62 + 12 × 6 + 32 = 36 + 72 + 32 = 140.
Therefore, the equilibrium price is 140.

Exercise

1. A manufacturer of tennis balls has a daily cost of 𝐶(𝑥) = 200 − 10𝑥 +


0.01𝑥 2 , where C is the total cost in dollars and x is the number of tennis
balls produced. What number of tennis balls will produce the minimum?
2. The height, h, in feet of an object above the ground is given by ℎ =
−16𝑡 2 + 64𝑡 + 199, 𝑡 ≥ 0 where t is the time in seconds. Find the time
it takes the object to strike the ground and find the maximum height of
the object.
3. The length of a rectangle is three more than twice the width. Determine
2
the dimensions that will give a total area of 27 m . What is the minimum
area that this rectangle can have?
4. The number of horsepower needed to overcome a wind drag on a certain
automobile is given by, 𝑁(𝑠) = 0.005𝑠 2 + 0.007𝑠 − 0.31, where s is the
speed of the car in miles per hour. How much horsepower is needed to
overcome the wind drag on this car if it is traveling 50 miles per hour? At
what speed will the car need to use 200 horsepower to overcome the wind
drag?

99
100 Unit 4: Types of Functions

4.4 The Exponential and


Logarithmic Functions
An important class of nonlinear functions that is of particular interest in
economics comprises the exponential and logarithmic functions.
These functions are useful for investigating problems associated with
economic growth and decay and mathematical problems in finance such
as the compounding of interest on an investment or the depreciation of an
asset.
Exponential functions are functions in which a constant base 𝑎 is raised
to a variable exponent 𝑥. The general form of an exponential function is
given by
𝑦 = 𝑎𝑥 , where 𝑎 > 0 and 𝑎 ≠ 1.
The parameter 𝑎 is known as the base of the exponential function. The
independent variable x occurs as the exponent of the base.

4.3.1 Exponential Functions


All exponential functions of the form 𝑓(𝑥) = 𝑎 𝑥 satisfy the following
properties:
Properties
1. The domain of 𝑓(𝑥) is the set of all real numbers; the range of 𝑓(𝑥) is the
set of all positive real numbers.
2. For all 𝑎 > 1, 𝑓(𝑥) is increasing; for 0 < 𝑎 < 1, 𝑓(𝑥) is decreasing.
3. For all 𝑎 > 0 with 𝑎 ≠ 1, 𝑓(0) = 1.
4. For 𝑎 > 1, 𝑓(𝑥) tends to 0 as x tends to −∞; for 0 < 𝑎 < 1, 𝑓(𝑥) tends
to 0 as x tends to +∞.
5. For 𝑎 > 1, 𝑓(𝑥) tends to +∞ (i.e., increases without bound) as x tends to
+∞; for 0 < 𝑎 < 1, 𝑓(𝑥) tends to +∞ as x tends to −∞.

In Figure on the left, the graphs


1 𝑥
of 𝑦 = 2𝑥 and 𝑦 = 2−𝑥 = (2)
are sketched for −4 ≤ x ≤ 4.
These graphs illustrate some of
the properties of exponential
functions.
BBA 120 Business Mathematics

Examples: Solve the equations


1 𝑒 2𝑥
a) 2𝑥 = 16 b) 5𝑥+3 5𝑥 = 54 c) 𝑒 4+𝑥 = 1
Solutions
1 𝑒 2𝑥
a) 2𝑥 = 16 b) 5𝑥+3 5𝑥 = 54 c) 𝑒 4+𝑥 = 1
1
2 𝑥 = 24 5𝑥+3+𝑥 = 54 𝑒 2𝑥−(4+𝑥) = 1

2𝑥 = 2−4 52𝑥+3 = 54 𝑒 𝑥−4 = 𝑒 0


equate indices equate indices equate indices
𝑥 = −4 2𝑥 + 3 = 4 𝑥−4 =0
1
𝑥 = = 0.5 𝑥=4
2

Example: Bacteria Growth


The number of bacteria present in a culture after t minutes is given by
4 𝑡
𝑁(𝑡) = 300 (3)
4 𝑡
Note that 𝑁 (𝑡) is a constant multiple of the exponential function (3) .
a. How many bacteria are present initially?
b. Approximately how many bacteria are present after 3 minutes?
Solution
a) Here we want to find N(t) when t = 0. We have
𝑁(02. 𝑥 = 813/4 = (811/4)3 = 33 = 27. )
4 0
= 300 ( ) = 300(1) = 300
3
Thus, 300 bacteria are initially present.

4 3 64 6400
𝑁(3) = 300 ( ) = 300 ( ) = ≈ 711
3 27 9

Hence, approximately 711 bacteria are present after 3 minutes.

4.3.2 Logarithmic Functions


Logarithmic functions are closely related to exponential functions and it
is this relationship that we will exploit in our description of some of their
key properties. Logarithms are useful for simplifying calculations
involving economic functions.
If we take the exponential function defined by 𝑦 = 𝑎 𝑥 and interchange
the dependent variable 𝑦 with the independent variable 𝑥, we obtain

10
1
102 Unit 4: Types of Functions

𝑥 = 𝑎𝑦 .
This defines a new function 𝑦 = log 𝑎 𝑥 , known as the logarithmic
function with base 𝑎, which is the exponent to which 𝑎 must be raised to
get x, i.e.,

A logarithm of 𝑥 = 𝑎 𝑦 ⇔ 𝑦 = log 𝑎 𝑥 .
a number is an Thus, the logarithmic function 𝑦 = log 𝑎 𝑥 is the inverse of the
exponent: exponential function 𝑦 = 𝑎 𝑥 .
log 𝑏 𝑥 is the There are two important bases:
power to which  a = 10 gives rise to common logarithms, written simply as log x.
we must raise b  a = e where e ≈ 2.71828 gives rise to natural logarithms, written
to get x. as ln x.
For example, Common and natural logarithms may be evaluated numerically by
log 2 8 = 3 pressing either the log or 𝑙𝑛 keys, respectively, on a scientific calculator.
because
23 = 8

Properties of the function 𝒇(𝒙) = 𝐥𝐨𝐠 𝒂 𝒙

1. The domain of the function is the set of all positive real numbers; the
range is the set of all real numbers.
2. For base a > 1, f(x) is increasing. For 0 < a < 1, f(x) is decreasing.
3. At x = 1, y = 0 independent of the base.

The graphs of the logarithmic functions


𝑦 = log 2 𝑥 and 𝑦 = log 1 𝑥 are shown
2
in Figure on the right.
These logarithmic functions may be
written equivalently as 𝑥 = 2𝑦 and
1 𝑦
𝑥 = (2) , respectively.
Note that these graphs are reflections of
the graphs of 𝑦 = 2𝑥 and 𝑦 = 2−𝑥 ,
respectively, in the line 𝑦 = 𝑥.

Examples
1. Evaluate the following:
a) log 8 64,
1
b) log 3 81,
c) log16 2 .
Solution.
BBA 120 Business Mathematics

a) Let 𝑦 = 𝑙𝑜𝑔8 64, then 8𝑦 = 64 = 82 and so 𝑦 = 2.


1 1
b) Let y = log 3 81, then 3𝑦 = 81 = 3−4 and so 𝑦 = −4.
c) Let y = log16 2, then 16𝑦 = 2 or (24 )𝑦 = 24𝑦 =
1
2 and so 4y = 1 and therefore 𝑦 = 4.

2. Solve the following for x:


a) log 4 𝑥 = 3,
3
b) log 81 𝑥 = 4

Solution.
a) 𝑥 = 43 = 64.
3 1 3
b) 𝑥 = 814 = (814 ) = 33 = 27.

Notation
The notation 𝑙𝑜𝑔(𝑥) is understood to be log10 (𝑥); for example, log(7)
means log10 (7). The notation 𝑙𝑛(𝑥) is understood to be log 𝑒 (𝑥); for
example, 𝑙𝑛(7) means log 𝑒 (7). Check on your calculator that 𝑙𝑜𝑔 12 =
1.0791812 and 𝐼𝑛 12 = 2.4849066.

Examples
1. Solve the equation 𝑙𝑛(𝑥 + 4)2 = 3 for x.

Solution.
2 𝑙𝑛(𝑥 + 4) = 3
3
𝑙𝑛(𝑥 + 4) =
2
𝑥 + 4 = 𝑒 1.5
𝑥 + 4 = 4.48169 to 5 decimal places
𝑥 = 0.48169 to 5 decimal places

2. Express log 𝑎 3 + log 𝑎 4 − log 𝑎 6 as a single logarithm.

10
3
104 Unit 4: Types of Functions

Solution.
log 𝑎 3 + log 𝑎 4 − log 𝑎 6 = 𝑙𝑜𝑔𝑎 (3 × 4) − 𝑙𝑜𝑔𝑎 6

3×4
= log 𝑎 ( )
6

= log 𝑎 2
3. Find the value of 𝑥 satisfying
log 𝑎 𝑥 = 3 log 𝑎 2 + log 𝑎 20 − log 𝑎 1.6
Solution.
log 𝑎 𝑥 = 3 log 𝑎 2 + log 𝑎 20 − log 𝑎 1.6

= log 𝑎 23 + log 𝑎 20 − log 𝑎 1.6

8 × 20
= 𝑙𝑜𝑔𝑎 ( )
1.6

= log 𝑎 100
Therefore 𝑥 = 100.

Unit 4: Types of Functions

4.4 The Exponential and


Logarithmic Functions

Examples
4. Solve the equation 𝑙𝑛(𝑥 + 4)2 = 3 for x.

Solution.
BBA 120 Business Mathematics

2 𝑙𝑛(𝑥 + 4) = 3
3
𝑙𝑛(𝑥 + 4) =
2
𝑥 + 4 = 𝑒 1.5
𝑥 + 4 = 4.48169 to 5 decimal places
𝑥 = 0.48169 to 5 decimal places

5. Express log 𝑎 3 + log 𝑎 4 − log 𝑎 6 as a single logarithm.

Solution.
log 𝑎 3 + log 𝑎 4 − log 𝑎 6 = 𝑙𝑜𝑔𝑎 (3 × 4) − 𝑙𝑜𝑔𝑎 6

3×4
= log 𝑎 ( )
6

= log 𝑎 2
6. Find the value of 𝑥 satisfying
log 𝑎 𝑥 = 3 log 𝑎 2 + log 𝑎 20 − log 𝑎 1.6
Solution.
log 𝑎 𝑥 = 3 log 𝑎 2 + log 𝑎 20 − log 𝑎 1.6

= log 𝑎 23 + log 𝑎 20 − log 𝑎 1.6

8 × 20
= 𝑙𝑜𝑔𝑎 ( )
1.6

= log 𝑎 100
Therefore 𝑥 = 100.

Unit 5: Mathematics of Finance

5.0 Simple and Compound Interest


Calculations
Today, businesses and individuals are faced with a bewildering array of
loan facilities and investment opportunities. In this section we explain
how these financial calculations are carried out to enable an informed
choice to be made between the various possibilities available.
We begin by considering what happens when a single lump sum is
invested and show how to calculate the amount accumulated over a
period of time.

10
5
106 Unit 5: Mathematics of Finance

5.0.1 Simple interest


Simple interest is a fixed percentage of the principal, 𝑃0 , that is paid to an
investor each year, irrespective of the number of years the principal has
been left on deposit; that is, money invested at simple interest will
increase in value by the same amount each year.
So, if the investor is paid a fixed annual amount, i% of 𝑃0 , then the
amount of simple interest, I, received over t years is given by the formula
𝐼 = 𝑃0 × 𝑖 × 𝑡
Therefore, the total value after t years, 𝑃𝑡 is the principal plus interest and
is given by
𝑃𝑡 = 𝑃0 + (𝑃0 × 𝑖 × 𝑡)
𝑃𝑡 = 𝑃0 (1 + 𝑖𝑡)
When the total value (future value), the interest rate and time are known,
the principal (present value) may be calculated by rewriting formula as
𝑃0 (1 + 𝑖𝑡) = 𝑃𝑡
𝑃𝑡
𝑃0 =
(1 + 𝑖𝑡)
Example
An investment of $3,000 is made at an annual simple interest rate of 5%.
How much additional money must be invested at an annual simple
interest rate of 9% so that the total annual interest earned is 7.5% of the
total investment?
Solution

I P i t

first (3,000)(0.05) = 150 3,000 0.05 1

additional 0.09 x x 0.09 1

total (3,000 + x)(0.075) 3,000 + x 0.075 1

First I fill in the P, i, and t columns with the given values.


Then I multiply across the rows (from the right to the left) in order to fill
in the I column.
Then add down the I column to get the equation
150 + 0.09 𝑥 = (3,000 + 𝑥)(0.075).
To find the solution, I would solve for the value of x.
Practice Exercise
1. How long does it take a principal of $25,000 at a simple interest rate of
5% to become $30,000?
BBA 120 Business Mathematics

2. $45,000 is deposited into a savings account. After one year, 4 months and
20 days it totals $52,500. Calculate the simple interest rate for this
account.
3. Determine the simple interest rate applied to a principal over 20 years if
the total interest paid equals the borrowed principal.
4. How long does it take a principal payment to triple at a simple interst rate
of 6%?
5. Find the total amount of simple interest that is paid over a perod of five
years on a principal of $ 30,000 at a simple interest rate of 6%.
6. Calculate the total worth of an investment after six months with a
principal of $10,000 at a simple interest rate of 3.5%.

5.0.2 Compound Interest


Under compound interest, at the end of each interest period, the interest
earned for that period is added to the principal (the invested amount) so
that it too earns interest over the next interest period. The basic formula
for the value (or compound amount) of an investment after n interest
periods' under compound interest is as follows:
𝑃𝑡 = 𝑃0 (1 + 𝑖)𝑡
The compound amount is also called the accumulated amount, and the
difference between the compound amount and the original principal, 𝑃𝑡 −
𝑃0 , is called the compound interest.

Example
Find the value, in 4 years’ time, of $10 000 invested at 5% interest
compounded annually.

10
7
108 Unit 5: Mathematics of Finance

Solution
In this problem, 𝑃0 = 10 000, 𝑖 = 5% and 𝑡 = 4, so the formula
𝑃𝑡 = 𝑃0 (1 + 𝑖)𝑡 gives
𝑃𝑡 = 10000(1 + 0.05)4
= 10000(1.05)4
= 12155.0
The compound interest formula derived above involves four variables, r,
t, 𝑃𝑡 and 𝑃0 . Provided that we know any three of these, we can use the
formula to determine the remaining variable. This is illustrated in the
following example.
Example
A principal of $25 000 is invested at 12% interest compounded annually.
After how many years will the investment first exceed $250 000?
Solution
We want to save a total of $250 000 starting with an initial investment of
$25 000. The problem is to determine the number of years required for
this on the assumption that the interest is fixed at 12% throughout this
time. The formula for compound interest is 𝑃𝑡 = 𝑃0 (1 + 𝑖)𝑡
We are given that
𝑃0 = 25 000, 𝑃𝑡 = 250 000, 𝑖 = 12
so we need to solve the equation
250 000 = 25 000 (1 + 0.12)𝑡

for n.
One way of doing this would just be to keep on guessing values of n until
we find the one that works.
However, a more mathematical approach is to use logarithms, because
we are being asked to solve an equation in which the unknown occurs as
a power. Following the method described in Section 2.3, we first divide
both sides by 25 000 to get
10 = (1.12)𝑛
Taking logarithms of both sides gives
𝑙𝑜𝑔(10) = 𝑙𝑜𝑔(1.12)𝑛
and if you apply rule 3 of logarithms you get
𝑙𝑜𝑔(10) = 𝑛 𝑙𝑜𝑔(1.12)
Hence
𝑙𝑜𝑔(10)
𝑛 =
𝑙𝑜𝑔(1.12)
1
=
0.049218023
BBA 120 Business Mathematics

= 20.3
Now we know that n must be a whole number because interest is only
added on at the end of each year. We assume that the first interest
payment occurs exactly 12 months after the initial investment and every
12 months thereafter.
The answer, 20.3, tells us that after only 20 years the amount is less than
$250 000, so we need to wait until 21 years have elapsed before it
exceeds this amount.

5.0.3 When interest is compounded several times per year


So far, it has been assumed that compound interest is compounded once a
year. In reality, interest may be compounded several times per year, for
example it may be compounded daily, weekly, monthly, quarterly, semi-
annually or continuously.
Each time period is known as a conversion period or interest period. The
number of conversion periods per year is denoted by the symbol, m; the
𝑖
interest rate applied at each conversion is .
𝑚

For example, an investment compounded twelve times per year will have
twelve conversion periods; therefore if a five-year investment was
compounded twelve times annually, then the investment would have sixty
conversion periods; that is,
𝑛 =𝑚×𝑡
where n = total number of conversion periods
m = conversion periods per year
t = number of years
The value of the investment at the end of n conversion periods is
𝑖 𝑛 𝑖 𝑚×𝑡
𝑃𝑡 = 𝑃0 (1 + ) = 𝑃0 (1 + )
𝑚 𝑚
Example
£5000 is invested for three years at 8% per annum compounded semi-
annually.
a) Calculate the total value of the investment.
b) Compare the return on the investment when interest is
compounded annually to that when compounded semi-annually.
c) Calculate the total value of the investment when compounded, (i)
monthly, (ii) daily. Assume all months consist of 365/12 days.
Solution
a) 𝑃0 = £5000, 𝑖 = 0.08, t = 3 and m = 2. Using Compound
interest formula, the total value after three years with n = m x t =
6 conversion periods is calculated as

10
9
110 Unit 5: Mathematics of Finance

0.08 (2)(3)
𝑃3 = 5000 (1 + )
2
= 5000(1 + 0.04)6
= 6326.59
b) The total value of £5000 after three years compounded annually
at 8% interest is
𝑃𝑡 = 5000(1 + 0.08)3
= 6298.56
When the same investment is compounded semi-annually, the
total value is £6326.59, a gain of £28.09 over the value when
compounded annually.
c) The value of the investment at the end of three years for monthly
and daily compounding is calculated as

i) Monthly compounding ii) Daily compounding


𝑖 𝑚𝑡 𝑖 𝑚𝑡
𝑃3 = 𝑃0 (1 + ) 𝑃3 = 𝑃0 (1 + )
𝑚 𝑚
0.08 (12)(3)
= 5000 (1 + ) 0.08 (365)(3)
12 = 5000 (1 + )
= 5000(1.2702) 365
= 5000(1.2712)
= 63151
= 63151

5.0.4 Annual percentage rate (APR) or Effective rate


Interest rates are usually cited as nominal rates of interest expressed as
per annum figures.
However, as compounding may occur several times during the year with
the nominal rate, the amount owed or accumulated will be different from
that calculated by compounding once a year.
So, a standard measure is needed to compare the amount earned (or
owed) at quoted nominal rates of interest when compounding is carried
out several times per year.
This standard measure is called the annual percentage rate (APR) or
effective annual rate. If interest is compounded once a year at the APR
rate, the investment would yield exactly the same return, 𝑃𝑡 , at the end of
t years, as it would if interest were compounded m times per year at the
nominal rate.
The formula for the annual percentage rate (APR) is given as follows:
𝑖 𝑚
𝐴𝑃𝑅 = (1 + ) − 1
𝑚
BBA 120 Business Mathematics

Example
1. What effective rate is equivalent to a nominal rate of 6% compounded (a)
semi-annually and (b) quarterly?
Solution
0.06 2
a) 𝐴𝑃𝑅 = (1 + ) − 1 = (1.03)2 − 1 = 0.0609 = 6.09%
2

0.06 4
b) 𝐴𝑃𝑅 = (1 + ) − 1 = (1.015)4 = 0.061364 = 6.14%
4

This example illustrates that, for a given nominal rate i, the effective rate
increases as the number of interest periods per year (n) increases.

2. If an investor has a choice of investing money at 6% compounded daily or


1
6 8 % quarterly, which is the better choice?
Solution
We determine the equivalent effective rate of interest for each nominal
rate and then compare our results.
The respective effective rates of interest are
0.06 365
𝐴𝑃𝑅 = (1 + ) − 1 ≈ 6.18%
365
And
0.06125 4
𝐴𝑃𝑅 = (1 + ) − 1 ≈ 6.27%
4
Since the second choice gives the higher effective rate, it is the better
choice (in spite of the fact that daily compounding may be
psychologically more appealing).

Practice Problems
1. A bank offers a return of 7% interest compounded annually. Find the
future value of a principal of $4500 after 6 years. What is the overall
percentage rise over this period?
2. Find the future value of $20 000 in 2 years’ time if compounded
quarterly at 8% interest.
3. Midwest Bank offers a return of 5% compounded annually for each
and every year. The rival BFB offers a return of 3% for the first year
and 7% in the second and subsequent years (both compounded
annually). Which bank would you choose to invest in if you decided
to invest a principal for (a) 2 years; (b) 3 years?
4. The value of an asset, currently priced at $100 000, is expected to
increase by 20% a year. (a) Find its value in 10 years’ time. (b) After
how many years will it be worth $1 million?

11
1
112 Unit 6: Matrix Algebra

5. How long will it take for a sum of money to double if it is invested at


5% interest compounded annually?
6. Find the future value of $100 compounded continuously at an
annual rate of 6% for 12 years.
7. How long will it take for a sum of money to triple in value if invested
at an annual rate of 3% compounded continuously?
8. If a piece of machinery depreciates continuously at an annual rate
of 4%, how many years will it take for the value of the machinery to
halve?
9. A department store has its own credit card facilities, for which it
charges interest at a rate of 2% each month. Explain briefly why this
is not the same as an annual rate of 24%. What is the annual
percentage rate?
10. Determine the APR if the nominal rate is 7% compounded
continuously.
11. Current annual consumption of energy is 78 billion units and this is
expected to rise at a fixed rate of 5.8% each year. The capacity of
the industry to supply energy is currently 104 billion units. (a)
Assuming that the supply remains steady, after how many years will
demand exceed supply? (b) What constant rate of growth of energy
production would be needed to satisfy demand for the next 50
years?

Unit 6: Matrix Algebra


ROWS ARE
6.0 Matrices
HORIZONTAL
AND
COLUMNS
6.0.1 Basic Definitions
ARE
VERTICAL. An m×n matrix is a rectangular array of real numbers with m rows and n
columns.
The numbers m and n are the dimensions of A.
The real numbers in the matrix are called its entries. The entry in row i
and column j is called aij or Aij.
To denote the entries in a matrix A of size n x m, say, we use the name of
the matrix, with double subscripts to indicate position, consistent with the
conventions above:
BBA 120 Business Mathematics

For the entry Al2 (read "A sub one-


two" or just "A one-two"), the first
subscript, 1, specifies the row and
the second subscript, 2, the column
in which the entry appears.
Similarly, the entry A23 (read "A
two-three") is the entry in the
second row and the third column.
Generalizing, we say that the
symbol Aij denotes the entry in the
ith row and jth column. In fact, a
matrix A is a function of two
variables with A(i,j) = Aij.
Example
Following is a 4×5 matrix with the entry A23 highlighted.
0 1 2 0 3
1 1
𝐴 = 3 −1 𝟏𝟎 3 2
3 1 0 1 −3
[2 1 0 0 1]
A matrix that has exactly one row, such as the 1 × 4 matrix
𝐴 = [1 7 12 3]
is called a row vector.
A matrix consisting of a single column; such as the 5 × 1 matrix
1
−2
15
9
[ 16 ]
is called a column vector.

6.0.2 Special Matrices


Certain types of matrices play important roles in matrix theory. We now
consider some of these special types.
Zero matrix
An m x n matrix whose entries are all 0 is called the 𝑚 × 𝑛 zero matrix
and is denoted by 𝑂𝑚×𝑛 or, more simply, by 0 if its size is understood.
Thus, the 2 x 3 zero matrix is
0 0 0
𝑂=[ ]
0 0 0
Square matrix
A matrix having the same number of columns as rows-for example, n
rows and n columns-is called a square matrix of order n. That is, an m x n
matrix is square if and only if m = n. For example, matrices

11
3
114 Unit 6: Matrix Algebra

4 6 −8
[ 6 2 0 ] and [5]
12 6 1
are square with orders 3 and ,1, respectively.
Identity matrix
Just as the zero matrix plays an important role as the identity in matrix
addition there is a special matrix, called the identity matrix that plays a
corresponding role in matrix multiplication:
The n x n identity matrix, denoted L" is the diagonal matrix whose main
diagonal entries are 1's.
For example, the identity matrices 𝐼3 and 𝐼4 are
1 0 0 0
1 0 0
0 1 0 0
𝐼3 = [0 1 0] 𝐼4 = [ ]
0 0 1 0
0 0 1
0 0 0 1

Example
A grocer sold 125 cans of tomato soup, 275 cans of beans, and 400 cans
of tuna. Write a row vector that gives the number of each item sold. If the
items sell for $0.95, $1.03, and $1.25 each, respectively, write this
information as a column vector.
Solution
Row Vector
Tomato Soup, Beans, Tuna
125 275 400
[125 275 400]
Column vector
BBA 120 Business Mathematics

0.95
[1.03]
1.25

6.0.3 Operations with Matrices


Transpose
The transpose, AT, of a matrix A is the matrix obtained from A by
writing its rows as columns. If A is an m×n matrix and B = AT, then B is
the n×m matrix with bij = aji.
Examples
1
0 1 2 𝑇 0
[1 ] =[ 3]
−1 10 1 −1
3
2 10

Sum, Difference
If A and B have the same dimensions, then their sum, A+B, is obtained
by adding corresponding entries. In symbols, (A+B)ij = Aij + Bij. If A and
B have the same dimensions, then their difference, A - B, is obtained by
subtracting corresponding entries. In symbols, (A-B)ij = Aij - Bij.
Properties of Matrix Addition
1. A+B=B+A commutative property
2. A + (B + C) = (A + B) + C associative property
3. A+O =A= O+A identity property

Scalar Multiple

If A is a matrix and c is a number (sometimes called a scalar in this


context), then the scalar multiple, cA, is obtained by multiplying every
entry in A by c. In symbols, (cA)ij = c(Aij).
Example
0 1 1 −1 2 −1
[1 ] + 2 [2 ] = [5 ]
−1 −2 −5
3 3 3

Properties of Scalar Multiplication

1. k(A + B) = kA + kB.
2. (k + l)A = kA + lA

11
5
116 Unit 6: Matrix Algebra

3. k(lA) = (kl)A
4. 0A = A
5. k0 = 0
We also have the following properties of the transpose operation,
where A and B are of the same size and k is any scalar:
 (𝐴 + 𝐵)𝑇 = 𝐴𝑇 + 𝐵𝑇
 (𝑘𝐴)𝑇 = 𝑘𝐴𝑇

Example: Demand Vectors for an Economy


Let matrix A represent the sales (in thousands of dollars) of a toy
company in 2007 in three cities, and let B represent the sales in the same
cities in 2009, where
𝐴𝑐𝑡𝑖𝑜𝑛 400 350 150
𝐴= [ ]
𝐸𝑑𝑢𝑐𝑎𝑡𝑖𝑜𝑛𝑎𝑙 450 280 850

𝐴𝑐𝑡𝑖𝑜𝑛 380 330 220


𝐵= [ ]
𝐸𝑑𝑢𝑐𝑎𝑡𝑖𝑜𝑛𝑎𝑙 460 320 750
If the company buys a competitor and doubles its 2009 sales in 2010,
what is the change in sales between 2007 and 2010?
Solution
Let the sales in 2010 be denoted by matrix C.
380 330 220 760 660 440
Then 𝐶 = 2𝐵 = 2 × [ ]= [ ]
460 320 750 920 640 1500
The change in sales between 2007 and 2012 is given by
760 660 440 400 350 150
𝐶−𝐴=[ ]−[ ]
920 640 1500 450 280 850
360 310 290
=[ ]
470 360 650

Matrix Multiplication
If A has dimensions m×n and B has dimensions n×p, then the product
AB is defined, and has dimensions m×p. The entry (AB)ij is obtained by
multiplying row i of A by column j of B, which is done by multiplying
corresponding entries together and then adding the results.
 Three points must be completely understood concerning this
definition of AB.
 First, the number of columns of A must be equal to the number
of rows of B.
 Second, the product AB has as many rows as A and as many
columns as B.
BBA 120 Business Mathematics

 Third, the definition refers to the product AB, in that order; A is


the left factor and B is the right factor. For AB, we say that B is
pre-multiplied by A or A is post-multiplied by B.

Examples
1. Let A be a 3 x 5 matrix and B be a 5 x 3 matrix. Then AB is
defined and is a 3 x 3 matrix. Moreover, BA is also defined and is
a 5 x 5 matrix.
2. If C is a 3 x 5 matrix and D is a 7 x 3 matrix, then CD is
undefined, but DC is defined and is a 7 x 5 matrix.
2
0 1 1 −1 −2
3
3. [ 1 ] [ 2
−1 3 −2 ] = [ 1 5 ]
3 − 3 3

4. Suppose that the prices (in dollars per unit) for products A, B, and
C are represented by the price vector
Price of
A B C
P = [2 3 4]
If the quantities (in units) of A, B, and C that are purchased are
given by the column vector
7 𝑈𝑛𝑖𝑡𝑠 𝑜𝑓 𝐴
𝑄 = [ 5 ] 𝑈𝑛𝑖𝑡𝑠 𝑜𝑓 𝐵
11 𝑈𝑛𝑖𝑡𝑠 𝑜𝑓 𝐶
then the total cost (in dollars) of the purchases is given by the
entry in the cost vector
7
𝑃𝑄 = [2 3 4] [ 5 ] = [(2 ∙ 7) + (3 ∙ 5) + (4 ∙ 11)] = [73]
11

6.0.4 Matrix Equations


Systems of linear equations can be represented by using matrix
multiplication. For example, consider the matrix equation

11
7
118 Unit 6: Matrix Algebra

𝑥1
1 4 −2 𝑥 4
[ ] [ 2] = [ ] (1)
2 −3 1 𝑥3 −3

The product on the left side has order 2 x 1 and hence is a column matrix.
Thus,
𝑥1 + 4𝑥2 − 2𝑥3 4
[ ]=[ ]
2𝑥1 − 3𝑥2 + 𝑥3 −3
By equality of matrices, corresponding entries must be equal, so we
obtain the system
𝑥1 + 4𝑥2 − 2𝑥3 = 4
2𝑥1 − 3𝑥2 + 𝑥3 = −3
Hence, this system of linear equations can be defined by matrix Equation
(1). We usually describe Equation (1) by saying that it has the form
𝑨𝑿 = 𝑩
where A is the matrix obtained from the coefficients of the variables, X is
a column matrix obtained from the variables, and B is a column matrix
obtained from the constants.
Matrix A is called the coefficient matrix for the system.
Example:
Write the system
2𝑥1 + 5𝑥2 = 4
{
8𝑥1 + 3𝑥2 = 7
in matrix form by using matrix multiplication.
Solution
2 5 x1 4
A=[ ] X = [x ] B=[ ]
8 3 2 7
then the given system is equivalent to the single matrix equation
𝑨𝑿 = 𝑩
2 5 𝑥1 4
[ ][ ] = [ ]
8 3 𝑥2 7

Exercises: Perform the indicated operations


4 2 −2 3 1 1 0
1. [3 10 0 ] [0 0 0 0]
1 0 2 0 1 0 1

0 1 1 0 1 0 1 0
2. [ ] ([ ] + [ ])
2 3 1 1 0 0 0 1

0 0 1 𝑥
3. [0 1 0] [𝑦]
1 0 0 𝑧
BBA 120 Business Mathematics

𝑎11 𝑎12 𝑥1
4. [𝑎 𝑎22 ] [𝑥 ]
21 2

5. A stockbroker sold a customer 200 shares of stock A, 300 shares


of stock B, 500 shares of stock C, and 25.0 shares of stock D. The
prices per share of A, B, C, and Dare $100, $150, $200, and $300,
respectively. Write a row vector representing the number of shares
of each stock bought. Write a column vector representing the price
per share of each stock. Using matrix multiplication, find the total
cost of the stocks.

11
9

You might also like